Professional Responsibility
Professional Responsibility
RESPONSIBILITY
PROFESSIONAL RESPONSIBILITY i.
PROFESSIONAL RESPONSIBILITY
TABLE OF CONTENTS
(MR02/CJC07)
ii. PROFESSIONAL RESPONSIBILITY
1. Separation of Funds . . . . . . . . . . . . . . . . . . . . . . . . . . . . . . . . 38
2. Recordkeeping Requirements . . . . . . . . . . . . . . . . . . . . . . . . . . 38
3. Lawyer’s Duty to Keep Client Informed . . . . . . . . . . . . . . . . . . . . . 38
4. Remittance of Funds and Property . . . . . . . . . . . . . . . . . . . . . . . . 38
5. Property Claimed by Lawyer and Other Person . . . . . . . . . . . . . . . . 38
PROFESSIONAL RESPONSIBILITY
1. State Regulation
a. State Legislatures
The relative powers of the legislature and courts of a state with respect to regulation
of the legal profession have never been clearly defined through legislation or case law.
It is generally assumed that the legislature establishes standards as aids to the judicial
branch, but that the courts may fashion more stringent rules.
b. State Courts
The inherent authority to regulate practice within the courts of a state is vested in the
highest court of the jurisdiction. Final decisionmaking power with respect to such
regulation is usually assumed by these courts.
2. Federal Regulation
a. Federal Courts
Each federal district and appellate court promulgates its own rules independent of those
that exist for the state courts in the jurisdiction where the federal court sits.
(MR02/CJC07)
2. PROFESSIONAL RESPONSIBILITY
1. Citizenship Requirements
A state may not require that an applicant be a citizen of the United States. [In re Griffiths,
413 U.S. 717 (1973)]
2. Residency Requirements
State residency requirements for bar admission violate the Privileges and Immunities Clause
of the United States Constitution. [Supreme Court of New Hampshire v. Piper, 470 U.S. 274
(1985)] Rationale: The right to practice law is a “fundamental” right. States may discrimi-
nate against nonresidents only where their reasons are substantial and the difference in
treatment bears a close or substantial relation to those reasons.
a. Admission on Motion
A state residency requirement for admission to the state’s bar without examination (on
motion) is unconstitutional. Requiring nonresidents to sit for a bar examination while
residents may be admitted on motion violates the Privileges and Immunities Clause.
[Supreme Court of Virginia v. Friedman, 487 U.S. 59 (1988)]
3. Character Requirements
Every jurisdiction places on each applicant the burden of showing that he possesses “good
moral character.”
a. Applicant’s Procedure
Each applicant is usually required to answer a detailed background questionnaire and
to furnish references. The refusal to furnish such information is grounds for rejection of
the applicant.
Example: An applicant who refuses to answer questions concerning his possible
membership in the Communist Party may be rejected because such
inquiries are merely a prelude to an investigation into his moral
character. [Konigsberg v. State Bar of California, 366 U.S. 36 (1961)]
b. Procedural Protections
An applicant is entitled to procedural due process with regard to actions taken by the
bar representatives reviewing his application. This includes the right to a hearing before
the bar committee and confrontation of adverse witnesses, as well as the right to
4. PROFESSIONAL RESPONSIBILITY
judicial review of the denial of an application based on bad moral character. [Willner v.
Committee on Character and Fitness, 373 U.S. 96 (1963)]
c. Conduct Failing the “Good Moral Character” Test
The purpose of the bar committee’s investigation into the background of an applicant
is to determine if there is anything in his past that reflects adversely upon his honesty
and integrity. The general benchmark used to measure this is whether any of the appli-
cant’s past conduct involved “moral turpitude,” i.e., acts that involve intentional dishon-
esty or are repugnant to accepted moral standards. Any conduct or charges against the
applicant involving such crimes as bribery, perjury, theft, murder, rape, etc., would fall
within the definition of “moral turpitude,” but a variety of illegal acts, such as draft
evasion and possession of marijuana for personal use, may or may not be acts involving
“moral turpitude,” depending upon the nature of the offense and the intent of the
individual.
Example: An applicant who had used aliases to hide his Jewish background from
employers and who had been arrested along with other discontented
laborers was found not to have committed an act of “moral turpitude.”
[Schware v. Board of Bar Examiners of New Mexico, supra]
d. Constitutional Limitations on Regulatory Authorities
Although a state regulatory agency may establish criteria that must be met by persons
seeking admission to practice law within the state, there must be a nexus between the
requirements for admission and an applicant’s fitness to practice law. For instance, the
state regulatory authority may not reject an applicant solely because of membership in
a political organization, as opposed to active and knowing participation in the organi-
zation with the intent to advance specific goals.
Nor may rejection be based on the personal beliefs of an applicant where those beliefs
are not translated into an illegal advocacy to action. In this context, the Supreme Court
rulings in bar admission proceedings have tracked the development of standards for the
invocation of free speech and associational rights under the First Amendment.
Example: An applicant may not be rejected merely on the basis of his membership
in the Communist Party where there is no showing that he ever engaged
in, or even advocated, actions to overthrow the government by force or
violence. [Schware v. Board of Bar Examiners of New Mexico, supra]
C. DISCIPLINARY PROCESS
the specific prohibitions in the Rules, lead to disciplinary measures. Under these prohibitions,
a lawyer may not:
(i) Violate or attempt to violate the Rules of Professional Conduct, knowingly assist or
induce another to do so, or use the acts of another to commit a violation;
(ii) Commit a criminal act that reflects adversely on the lawyer’s honesty, trustworthiness,
or fitness as a lawyer in other respects;
(vi) Knowingly assist a judge or judicial officer in conduct that violates applicable rules of
judicial conduct or other law.
[RPC 8.4]
2. Disciplinary Procedure
While the details of disciplinary procedure vary from state to state, there is a general proce-
dural scheme along the following lines:
a. Complaint
The disciplinary proceeding is initiated by a complaint to the state bar, which may be
filed by a dissatisfied client, a member of the bar, or any other interested party. The
filing of a complaint is generally considered a privileged communication and the
complainant is usually protected from subsequent action by the attorney for defamation
or malicious prosecution as a matter of public policy.
1) Commencement of Hearing
If a decision is made to pursue the complaint, a nonpublic hearing will usually be
held before the grievance committee after notice to the attorney under investiga-
tion.
2) Procedural Protections
The Due Process Clauses of the Fourteenth Amendment and relevant state consti-
tutions are applicable to state disciplinary proceedings. Thus, while the specific
procedures vary by jurisdiction, a lawyer subject to a disciplinary hearing retains
the basic rights to counsel and to cross-examination of witnesses at the hearing.
[In re Ruffalo, 390 U.S. 544 (1968)] He also has the absolute right to invoke,
without penalty, the Fifth Amendment protection against self-incrimination
under the Supreme Court’s ruling in Spevack v. Klein, 385 U.S. 511 (1967).
6. PROFESSIONAL RESPONSIBILITY
a. Choice of Law
A tribunal considering conduct connected with a matter pending before it will apply
the Rules of Professional Conduct of the state in which it sits, unless the tribunal’s rules
provide otherwise. Regarding any other conduct, the tribunal will apply: (i) the rules of
the state in which the conduct occurs; or (ii) the rules of another state in which occurred
the predominant effect of the conduct. A safe harbor is provided for a lawyer whose
conduct conforms to the rules of a state in which the lawyer reasonably believes the
predominant effect will occur. [RPC 8.5(b)] Thus, if the lawyer is reasonably mistaken
about the place of predominant effect, she will not be subject to discipline under the
rules of the state where the predominant effect actually occurs. Regarding conflicts
of interest only, a lawyer and client may enter into a written agreement specifying the
“predominant effect” jurisdiction—e.g., “Client and Lawyer anticipate that the predom-
inant effect of the lawyer’s conduct will be in Ohio and agree that the Ohio Rules
of Professional Conduct and Ohio law relating to conflicts of interest should govern
Lawyer’s work on this matter.” [RPC 8.5, comment 5]
4) Estate Planners
Estate planners may not draft wills or prepare plans for specific individuals
but may disseminate general information to the public through commercial or
noncommercial means.
Example: An estate planner published a book entitled How to Avoid Probate,
which was in effect a “do-it-yourself” probate kit. He was subse-
quently charged with practicing law without a license. The court
ruled that he was merely disseminating information to the public
to enable individuals to represent themselves. [New York County
Lawyers’ Association v. Dacey, 287 N.Y.S.2d 422 (1967)]
b. Representation of Clients
1) Corporations
Corporations, other than nonprofit legal service groups or recognized professional
corporations, generally may not represent a client directly in judicial proceedings
or provide representation through their attorney-employees.
8. PROFESSIONAL RESPONSIBILITY
2) Collection Agencies
Collection agencies in a number of jurisdictions may not bring suit directly to
enforce a debt assigned to them for collection. However, a state statute defining
debt collection as the practice of law, thus limiting debt collection activities to
members of that state’s bar, was held to be an unconstitutional imposition on inter-
state commerce. [National Revenue Corp. v. Violet, 807 F.2d 285 (1st Cir. 1986)]
4. Multi-Jurisdictional Practice
A lawyer who is not licensed to practice in a state must not: (i) open a law office in that state,
(ii) represent that she is admitted to practice in that state, or (iii) establish a “systematic and
continuous presence” in that state without becoming licensed there. [RPC 5.5]
A. FORMS OF PRACTICE
Lawyers have the opportunity to practice law in myriad ways. The most prevalent of these various
forms are sole practitioners; law firms; lawyers involved in the executive, legislative, and judicial
aspects of government; legal services organizations; and counsel to special interest groups and
business enterprises, most notably corporations, banks, and insurance companies.
1. Law Firms
in which the lawyer is not actively and regularly practicing with the firm. [RPC
7.5(c)]
3. Legal Corporations
Due to favorable tax treatment for corporate pension and profit-sharing plans, all 50 states
and the District of Columbia allow lawyers to incorporate into highly regulated special
business structures known by various titles such as professional corporations and profes-
sional associations. The liability of the lawyer to the client may not be limited in a legal
corporation.
(i) If participating in the decision would be incompatible with the lawyer’s obligations
to a client under RPC 1.7 (relating to conflicts of interest); or
(ii) Where the decision could have a material adverse effect on the representation of a
client of the organization whose interests are adverse to a client of the lawyer.
[RPC 6.3]
5.1(b)] A lawyer having direct supervisory authority over a nonlawyer must make reasonable
efforts to ensure that the person’s conduct is compatible with the professional obligations of
the lawyer. [RPC 5.3(b)] This responsibility applies to nonlawyers within the firm (e.g., legal
assistants) and outside the firm (e.g., document management companies, private investiga-
tors).
layperson, initiated by a lawyer (or lawyer’s agent), that is designed to entice the layperson to
hire the lawyer. A blatant form of solicitation is “ambulance chasing,” in which a lawyer (or
lawyer’s agent) seeks out injured people and urges them to hire the lawyer to represent them.
(iii) Are narrowly tailored to serve the substantial interest (i.e., the means chosen reason-
ably fit the ends sought).
[Florida Bar v. Went For It, Inc., 515 U.S. 618 (1995)—citing Central Hudson Gas & Electric
Corp. v. Public Service Commission of New York, 447 U.S. 557 (1980)] Moreover, the
government may require attorneys to make disclosures to prevent misleading advertisements
as long as the disclosures are not unduly burdensome and are reasonably related to the state’s
interest in preventing deception. [Milavetz, Gallop & Milavetz, P.A. v. United States, 559
U.S. 229 (2010)—lawyers (and other “debt relief agencies”) could be required to include
information about their legal status and the nature of the assistance provided, as well as the
possibility of the debtor’s filing for bankruptcy]
Example: After conducting a two-year study on the effect of lawyer advertising on
public opinion, Florida adopted a rule prohibiting lawyers from sending
targeted, direct-mail solicitations to victims and their relatives for 30 days
following an accident or disaster. The regulation was upheld: (i) the state has
a substantial interest in protecting the privacy and tranquility of its citizens
as well as in protecting the reputation of the legal profession; (ii) the study
showed that the public was offended by targeted solicitations sent shortly after
an accident or disaster and that the 30-day ban directly advances the state
interests; and (iii) the rule is narrowly tailored (30-day ban) to achieve the
desired results. [Florida Bar v. Went For It, Inc., supra]
Compare: 1) Missouri could not prohibit lawyers from advertising the jurisdictions in
which they are licensed nor prohibit the general mailing of cards announcing
the opening of an office, as there was no showing that these advertisements
were misleading or difficult to supervise. [In re R.M.J., 455 U.S. 191 (1982)]
3) Kentucky could not flatly prohibit lawyers from sending letters to poten-
tial clients faced with particular legal problems. [Shapero v. Kentucky Bar
Association, 486 U.S. 466 (1988)—lawyer could not be prohibited from
sending truthful, nondeceptive letters to people facing foreclosure]
a. In-Person Solicitation May Be Banned
The Supreme Court has found that in-person solicitation is likely to result in
overreaching or misleading a layperson. Therefore, states may adopt prophylactic rules
to forbid in-person solicitation for profit. [Ohralik v. Ohio State Bar Association, 436
U.S. 447 (1978)]
B. ADVERTISING
1. Basic Rule—Communications Must Be True and Not Misleading
A lawyer must not make a false or misleading communication about the lawyer or the
lawyer’s services. “False or misleading” includes material misrepresentations of law or fact,
and omissions of fact that are necessary to make a statement, considered as a whole, not
materially misleading. [RPC 7.1]
2. Truthful but Misleading Communications
A truthful communication is misleading if: (i) there is a substantial likelihood that the
communication will lead a reasonable person to reach a conclusion about the lawyer or her
services for which there is no reasonable foundation; (ii) it reports the lawyer’s achievements
on behalf of clients and is presented in a manner that would lead a reasonable person to form
an unjustified expectation of receiving the same results in a similar matter, without refer-
ence to the specific circumstances of each client’s case; or (iii) it contains an unsubstanti-
ated comparison of the lawyer’s services or fees with those of other lawyers and is presented
with such specificity as to lead a reasonable person to conclude that the comparison can be
substantiated. [RPC 7.1, comments 2, 3]
3. Limits on Advertising
RPC 7.2(a) gives lawyers broad latitude in advertising their services in a true and nonmis-
leading manner. They may advertise in written, recorded, or electronic communication,
including public media. However, even true, nonmisleading advertisements are subject to the
following limits:
a. Identification of Advertiser
An advertisement must contain the name and office address of at least one lawyer or
firm responsible for its content. [RPC 7.2(c)]
b. Payments for Recommending a Lawyer’s Services
A lawyer must not give anything of value for a person’s recommendation of the
lawyer’s services. However, a lawyer may:
(i) Pay the reasonable costs of permitted advertisements (e.g., broadcast airtime,
directory listings, or newspaper ads);
(ii) Pay the usual charges of a legal service plan or a not-for-profit or qualified lawyer
referral service, i.e., one that has been approved by an appropriate regulatory
authority;
PROFESSIONAL RESPONSIBILITY 17.
(iii) Pay for purchase of a law practice pursuant to RPC 1.17 (see II.E., supra); or
[RPC 7.2(b)] If a lawyer accepts assignments or referrals from a legal service plan or
a lawyer referral service, she must act reasonably to assure that the activities of the
plan or service are compatible with her professional obligations; e.g., a communication
with prospective clients by the plan or service must not be false or misleading. [RPC
7.2, comment 7] Similarly, a lawyer may pay others for generating client leads (e.g., a
bankruptcy lawyer may pay to be listed on an informational bankruptcy website that
offers to connect each visitor with a lawyer) but the lead generator’s communications
must not be false or misleading, and the lead generator must not recommend the lawyer.
[RPC 7.2, comment 5]
C. SOLICITATION
A solicitation is a targeted communication initiated by the lawyer that is directed to a specific
person and that offers to provide, or can reasonably be understood as offering to provide, legal
services. However, a communication generally is not a solicitation if it: (i) is directed to the
general public (e.g., through a billboard, website, television commercial, or Internet banner
advertisement); (ii) responds to a request for information; or (iii) is automatically generated in
response to an Internet search.
a. Exceptions
(i) The target of the solicitation has made known to the lawyer that she does not want to be
solicited by the lawyer; or
[RPC 7.3(b)] Note that federal law prohibits lawyers from communicating with victims of an
airplane accident, or their families, until 45 days after the accident. [49 U.S.C. §1136(g)(2)]
Admiralty,” or a substantially similar designation. A lawyer may not state or imply that she is
certified as a specialist in a particular field of law, unless:
(i) The lawyer has been certified as a specialist by an organization approved by an appropriate
state authority or accredited by the ABA; and
(ii) The name of the certifying organization is clearly identified in the communication.
[RPC 7.4]
(i) A person manifests an intent that the lawyer provide legal services and the lawyer agrees;
(ii) A person manifests an intent to have the lawyer represent him, the lawyer fails to make clear
that he does not want to undertake the representation, and the lawyer knows or should know
that the prospective client is reasonably relying on the lawyer to provide the services; or
2) Client Casey calls lawyer Lisa’s office asking that Lisa represent him
in a court proceeding relating to his arrest for driving under the influence
(“DUI”). Lisa is out of the office. Casey tells Lisa’s secretary that he under-
stands that Lisa handles many DUI cases and hopes that she will take the case
even though the court date is only 10 days away. The secretary tells Casey
to send over all papers relevant to the proceeding. She does not tell him that
Lisa will decide whether to take the case only after reviewing the papers. One
day before Casey’s court date, Lisa phones Casey and declines to represent
him. Here, it would likely be found that an attorney-client relationship existed
because Casey’s reliance was reasonable. Lisa regularly handled DUI cases,
20. PROFESSIONAL RESPONSIBILITY
her agent responded to his request for help by asking him to send the papers,
and the imminence of the hearing made it appropriate for Lisa to decline
while there was still time for Casey to get another lawyer. [Restatement §14,
illus. 4]
C. ACCEPTING APPOINTMENTS
A lawyer must not seek to avoid appointment by a tribunal to represent a person except for good
cause, such as: (i) representing the client is likely to result in a violation of the RPC or other law;
(ii) representing the client is likely to result in an unreasonable financial burden on the lawyer; or
(iii) the client or the cause is so repugnant to the lawyer as to be likely to impair the lawyer-client
relationship or the lawyer’s ability to represent the client. [RPC 6.2]
(i) The lawyer reasonably believes that he can competently and diligently represent
each affected client, despite the concurrent conflict;
(ii) The representation is not prohibited by law (e.g., a state statute prohibits the
lawyer from switching from the prosecution side to the defense side of a criminal
case);
(iii) The representation does not involve the assertion of a claim by one client against
another client who is represented by the lawyer in the same case pending before a
court or other tribunal; and
[RPC 1.7(b)] Note that condition (i), above, creates a “reasonable lawyer” standard—
i.e., if a reasonable lawyer looking at the facts would have to advise an affected client
not to consent, then the conflict is “unconsentable,” and client consent will not resolve
the conflict. [RPC 1.7, comment 14] Note also that the rule requires “informed” consent,
22. PROFESSIONAL RESPONSIBILITY
which means that each affected client must understand how the conflict could harm
her. [RPC 1.7, comment 18] Sometimes a lawyer’s duty of confidentiality to one client
prevents him from disclosing information that another client needs in order to under-
stand the conflict of interest. In this situation, the conflict is unconsentable. [RPC 1.7,
comment 19] Consent also must be “confirmed in writing,” which means either of two
things: (i) the client executes a writing, or (ii) the lawyer promptly records and trans-
mits a confirmatory writing to the client after the client orally consents to the conflict.
Under the RPC, a “writing” includes electronic transmissions (e.g., e-mail). [RPC 1.7,
comment 20]
1) Criminal Litigation
The Sixth Amendment guarantees every criminal defendant the right to effec-
tive assistance of counsel. Because the interests of criminal co-defendants are
very likely to diverge, ordinarily a lawyer should not try to defend two people in a
criminal case. [RPC 1.7, comment 23]
2) Civil Matters
In civil litigation and in nonlitigation matters, one lawyer may represent two
parties or clients whose interests are potentially in conflict if, after analyzing the
facts and applicable law, the lawyer concludes that she can effectively represent
both clients. (If the clients’ positions are already antagonistic, the lawyer gener-
ally should not take on the representation.) The lawyer must obtain the clients’
consent as described in a., above. In doing so, the clients must be advised that if
litigation arises between them regarding the matter, neither of them can claim the
attorney-client privilege for their communications with that lawyer. Therefore, the
lawyer should ordinarily make clear to all clients at the outset that whatever one
client discloses will be shared with all of the other clients. If the potential conflict
eventually ripens into a present conflict, the lawyer must analyze the situation
PROFESSIONAL RESPONSIBILITY 23.
again and re-address the conflict, which may involve repeating the steps in a.,
above. However, if a reasonable lawyer would have to advise either of the two
clients not to consent, the lawyer must withdraw. [RPC 1.7 and comments] The
lawyer may continue to represent one consenting client, but only if the client who
is dropped provides informed consent, confirmed in writing. [See RPC 1.9(a)]
c. Specific Kinds of Conflicts
RPC 1.8 augments the foregoing general rule on concurrent conflicts by identifying
specific conflict situations. As you will see, some of the situations can be solved by
obtaining the affected clients’ informed consents, and others cannot.
(i) The transaction and terms under which the lawyer acquires the interest are
fair and reasonable to the client and are fully disclosed and transmitted in
writing to the client in a manner that can be reasonably understood by the
client;
(ii) The client is informed in writing of the desirability of seeking, and is given
a reasonable chance to seek, the advice of independent legal counsel on the
transaction; and
(iii) The client understands the lawyer’s role in the transaction, including whether
the lawyer is representing the client, and the client gives informed consent in
a signed writing.
[RPC 1.8(a)]
grandparent, or other person with whom the lawyer or client has a close, familial
relationship.
a. Consultation
Whether communications constitute a consultation making a person a prospective client
depends on the circumstances. It is likely that a consultation has occurred when:
However, a consultation does not occur when a person provides information to a lawyer
in response to an advertisement that merely consists of the lawyer’s biographical infor-
mation, areas of practice, contact information, or legal information of general interest.
Furthermore, someone who contacts the attorney in an effort to disqualify the attorney
(i.e., by creating a conflict of interest) is not a prospective client. [RPC 1.18, comment 2]
(i) Demonstrate that the lawyer who held consultations with the prospective client
took care to avoid exposure to more confidential information than was neces-
sary to determine whether to represent the prospective client;
(ii) Demonstrate that the disqualified lawyer is timely screened from any participa-
tion in the matter and will not share the fee; and
[RPC 1.18(d)(2)]
a. Uncontested Issue
If the attorney’s testimony will relate solely to an uncontested issue, representation is
not prohibited.
Example: In a proceeding concerning an uncontested will, an attorney may testify
as to the testamentary capacity of the decedent.
PROFESSIONAL RESPONSIBILITY 27.
c. Substantial Hardship
If the withdrawal of the attorney for the purpose of testifying would result in substantial
hardship to the client (e.g., because of the distinctive value of the attorney or firm to the
case), the attorney may continue to represent the client.
Example: X represents B in a highly complex case that is in the middle of trial.
Defendant C attempts to bribe X. X’s testimony becomes highly relevant
to the case at this point, but he cannot withdraw from the case because B
would not be able to secure adequate replacement representation on such
notice. X should be permitted to testify and to continue to represent B.
(see V.B.2., infra). [RPC 1.13(c)] However, the lawyer’s authority to report to
outsiders applies only if, and to the extent that, the lawyer reasonably believes
that reporting is necessary to prevent substantial injury to the organization. The
authority to report to outsiders does not apply to a lawyer who is hired by the
organization to investigate an alleged violation of law or to defend the organization
or its constituents against a claimed violation of law. [RPC 1.13(d)]
Example: Attorney A’s corporate client C produces frozen chicken pies. C’s
production process creates large quantities of liquid waste, which C
is supposed to pump into recycling tanks. C’s manufacturing vice
president sometimes orders his workers to dump the waste into
a ditch that drains into some neighboring wetlands; the dumping
is cheaper and quicker, but it gradually destroys the wetlands in
violation of state and federal environmental laws. When A learned
about the dumping, she reported it to C’s president and warned
him that C will be fined millions of dollars if it gets caught. C’s
president ignored A’s warning, so A reported the matter to the
highest authority in the company—the audit committee of the
board of directors. The audit committee did nothing. If A reason-
ably believes that the company will be substantially injured if the
dumping continues, A may report the relevant information to the
appropriate environmental enforcement authority, even if some of
that information would otherwise be protected by the duty of confi-
dentiality. [See RPC 1.13(c)]
3) Whistleblower Protection
A lawyer who reasonably believes that she has been fired because of actions
she took pursuant to RPC 1.13(b) or (c), or who withdraws when circumstances
require or permit her to do so, must proceed as she reasonably believes necessary
to inform the organization’s highest authority of her firing or withdrawal. [RPC
1.13(e)]
violating federal or state securities laws or similar laws. The SEC rules [see 17 C.F.R.
§§205.1 et seq.] apply to “securities lawyers,” meaning not only lawyers who practice
before the SEC, but also lawyers who advise clients about documents that will be filed
with the SEC or who advise clients about whether information must be filed with the
SEC.
6. Imputation of Conflicts
(i) The disqualified lawyer is timely screened from any participation in the matter and
is apportioned no part of the fee therefrom; and
PROFESSIONAL RESPONSIBILITY 31.
(ii) Written notice is promptly given to the appropriate government agency to enable it
to ascertain compliance with the provisions of this rule.
[RPC 1.11(b)]
(iv) The likelihood, if apparent to the client, that acceptance of the particular employment
will preclude other employment;
(v) The customary fee within the locality for similar work;
(vii) The nature and length of the relationship between the parties;
(ix) The time limitations imposed by the client or the circumstances; and
[RPC 1.5(a)]
PROFESSIONAL RESPONSIBILITY 33.
4. Contingent Fees
A contingent fee is a fee that is dependent on the successful resolution of a client’s case and
payable from the judgment proceeds. Usually, such fees take the form of a set percentage of
the recovery, the fee being zero if there is no recovery. The RPC approve of contingent fees
within certain limits.
a. Disclosure Requirements
A contingent fee agreement must be in writing, signed by the client, and state the
method by which the fee is to be determined, including: (i) the percentage or percent-
ages that accrue to the lawyer in the event of settlement, trial, or appeal; (ii) litigation
and other expenses to be deducted from the recovery; and (iii) whether such expenses
are to be deducted before or after the contingent fee is calculated. Also, the agreement
must clearly notify the client of any expenses for which the client will be liable regard-
less of whether the client prevails. Upon conclusion of a contingent fee matter, the
lawyer must provide the client with a written statement of the outcome of the matter.
Such statement must also show any remittance to the client and the method of its deter-
mination. [RPC 1.5(c)]
1) Criminal Cases
A lawyer is subject to discipline for using a contingent fee arrangement when
defending a person in a criminal case. [RPC 1.5(d)(2)]
34. PROFESSIONAL RESPONSIBILITY
1. Competent Representation
Competent representation requires the legal knowledge, skill, thoroughness, and preparation
reasonably necessary for the representation. [RPC 1.1]
a client on a particular matter, they should consult with each other and with the
client regarding the scope of their respective representations and the allocation of
responsibility among them. [RPC 1.1, comment 7]
2. Diligence
A lawyer must act with reasonable diligence and promptness in representing a client. [RPC
1.3] A lawyer’s workload must be controlled so that each matter can be handled competently.
[RPC 1.3, comment 2]
3. Communication
A lawyer must keep a client reasonably informed about the status of a matter and promptly
comply with reasonable requests for information. A lawyer must explain a matter to the
extent reasonably necessary to permit the client to make informed decisions regarding the
representation. When the lawyer knows that the client expects assistance not permitted by the
RPC or other law, the lawyer must consult with the client about any relevant limitation on the
lawyer’s conduct. [RPC 1.4]
1. Attorney-Client Privilege
Generally, the attorney-client privilege allows a client to refuse to testify and prevent his
attorney from testifying in court about communications between the two. It applies to confi-
dential communications made by an individual to an attorney who is sought out for the
purpose of obtaining legal advice. The theory behind the rule is to promote full disclosure
between the client and the attorney so that the attorney can most capably represent the client.
A client is a person or entity seeking legal services from an attorney.
(i) The employee communicates with the attorney at the direction of the employee’s
superior;
(ii) The employee knows that the communication is to help the corporation get legal
advice; and
(iii) The communication concerns a subject within the scope of the employee’s duties
for the corporation.
c. Duration of Privilege
The attorney-client privilege continues indefinitely. Termination of the attorney-client
relationship does not terminate the privilege. The attorney-client privilege even
survives the death of the client. Rationale: Knowing that communications will remain
confidential even after death encourages the client to communicate fully and frankly
with his attorney. [Swidler & Berlin v. United States, 524 U.S. 399 (1998)]
2. Duty of Confidentiality
Except as noted below, a lawyer must not reveal information relating to the representation
of a client. [RPC 1.6] The duty of confidentiality applies to all information “relating to the
representation,” regardless of when or where it was acquired, whether the client asked for
it to be kept in confidence, and whether revealing it might harm or embarrass the client.
The duty of confidentiality continues to apply even after the lawyer-client relationship has
terminated. Note that the duty of confidentiality is broader than the attorney-client privilege,
which covers only confidential communications between the attorney and the client, or their
respective agents. Note also that an attorney must not make disclosures that could reasonably
lead to discovery of confidential information by a third person.
a. Exceptions to Duty
order or other law requiring the lawyer to give information about a client. Note
that there are also circumstances under which the ethical rules permit or require
the lawyer to disclose information relating to the representation (e.g., when false
evidence is introduced in court). Lastly, there may be statutes requiring the lawyer
to reveal information in certain circumstances. [RPC 1.6]
5) Disclosure Necessary to Collect a Fee or Protect Lawyer
A lawyer may reveal information to the extent the lawyer reasonably believes
necessary to establish a claim or defense on behalf of the lawyer in a controversy
between the lawyer and the client, to establish a defense to a criminal charge or
civil claim against the lawyer based upon conduct in which the client was involved,
or to respond to allegations in any proceeding concerning the lawyer’s representa-
tion of the client. [RPC 1.6(b)(5)]
(i) The lawyer reasonably believes that making the evaluation is compatible with other
aspects of the lawyer’s relationship with the client; and
(ii) The lawyer obtains the client’s informed consent if the evaluation is likely to materi-
ally harm the client’s interests.
[RPC 2.3] Such evaluation may include, for example, an opinion concerning the title of
property rendered at the behest of a vendor for the information of a prospective purchaser, or
at the behest of a borrower for the information of a prospective lender. In some situations, the
evaluation may be required by a government agency.
38. PROFESSIONAL RESPONSIBILITY
1. Separation of Funds
A lawyer must hold property of clients or third persons that is in the lawyer’s possession in
connection with a representation separate from the lawyer’s own property. Funds must be
kept in a separate account maintained in the state where the lawyer’s office is situated, or
elsewhere with the consent of the client or third person. Other property must be identified as
such and appropriately safeguarded. [RPC 1.15(a)] A lawyer may deposit her own funds in a
client trust account solely to pay bank service charges, but only in an amount necessary for
this purpose. A lawyer must deposit into the account legal fees and expenses that were paid
in advance, to be withdrawn only as fees are earned or expenses are incurred. [RPC 1.15(b),
(c)]
2. Recordkeeping Requirements
Complete records of client account funds and other property must be kept by the lawyer and
must be preserved for a period to be designated by individual states (typically five years)
after termination of the representation. [RPC 1.15(a)]
A. DUTY AS ADVISER
In representing a client, a lawyer must exercise independent professional judgment and render
candid advice. In rendering advice, a lawyer may refer not only to law, but to other considerations
such as moral, economic, social, and political factors that may be relevant to the client’s situation.
[RPC 2.1]
B. SCOPE OF REPRESENTATION
PROFESSIONAL RESPONSIBILITY 39.
(i) A person who supervises, directs, or regularly consults with the organization’s
lawyer concerning the matter;
(ii) A person with authority to obligate the organization with respect to the matter; or
(iii) A person whose act or omission in connection with the matter may be imputed to
the organization for purposes of civil or criminal liability.
However, if the person is represented in the matter by her own counsel, then consent by
that counsel (rather than the organization’s counsel) is sufficient. [RPC 4.2, comment 7]
Note that consent is usually not needed before talking with a former employee, unless
the former employee has extensive knowledge of the organization’s relevant privileged
information. [ABA Formal Op. 91-359 (1991); Camden v. Maryland, 910 F. Supp. 1115
(D. Md. 1996)]
lawyer’s role in the matter, the lawyer must make reasonable efforts to correct the
misunderstanding. Moreover, if the lawyer knows or reasonably should know that the
interests of the unrepresented person are or have a reasonable possibility of being in
conflict with the interests of the lawyer’s client, the lawyer may only advise the unrepre-
sented person to obtain counsel. No other legal advice is permitted. [RPC 4.3]
2. Respect for Rights of Third Persons
In representing a client, a lawyer must not use means that have no substantial purpose other
than to embarrass, delay, or burden a third person; or use methods of obtaining evidence that
violate the legal rights of such a person. [RPC 4.4]
2. Expediting Litigation
A lawyer must make reasonable efforts to expedite litigation consistent with the interests
42. PROFESSIONAL RESPONSIBILITY
of the client. [RPC 3.2] Delay should not be indulged merely for the convenience of the
advocates or for the purpose of frustrating an opposing party’s attempts to obtain rightful
redress.
(ii) Fail to correct a false statement of material fact or law previously made to the tribunal by
the lawyer;
(iii) Fail to disclose to the tribunal legal authority in the controlling jurisdiction known to the
lawyer to be directly adverse to the position of the client and not disclosed by opposing
counsel; or
(iv) Offer false evidence. If a lawyer has offered material evidence and the lawyer comes to
know of its falsity, the lawyer must take reasonable remedial measures, including, if neces-
sary, disclosure to the tribunal.
These four duties continue to the conclusion of the proceeding—i.e., when a final judgment has
been affirmed on appeal or the time for taking an appeal has passed.
1. False Evidence
a. General Rules
The following general rules apply in civil and criminal cases, and in all other kinds of
proceedings that are pending before a tribunal, such as binding arbitration or an admin-
istrative agency proceeding when the agency will adjudicate the rights of the parties.
These general rules also apply in ancillary proceedings, such as a deposition. The sole
exception to these general rules is when a criminal defendant insists on testifying falsely
(see b., below).
(i) Unlawfully obstruct another party’s access to evidence or unlawfully alter, destroy, or
conceal a document or other material having potential evidentiary value, or counsel or assist
another person to do any such act;
(ii) Falsify evidence, counsel or assist a witness to testify falsely, or offer an inducement to a
witness that is prohibited by law;
(iii) Knowingly disobey an obligation under the rules of a tribunal except for an open refusal
based on an assertion that no valid obligation exists;
(iv) In pretrial procedure, make frivolous discovery requests or fail to make reasonably diligent
efforts to comply with legally proper discovery requests by an opposing party;
(v) In trial, allude to any matter that the lawyer does not reasonably believe is relevant or that
will not be supported by admissible evidence, assert personal knowledge of facts in issue
except when testifying as a witness, or state a personal opinion as to the justness of a cause,
the credibility of a witness, the culpability of a civil litigant, or the guilt or innocence of an
accused; or
(vi) Request a person other than a client to refrain from voluntarily giving relevant informa-
tion to another party unless: (i) the person is a relative or an employee or other agent of a
client; and (ii) the lawyer reasonably believes that the person’s interests will not be adversely
affected by refraining from giving such information.
[RPC 3.4]
I. TRIAL PUBLICITY
To maintain the dignity of the profession and ensure against prejudicial publicity, the RPC provide
restraints on public statements by lawyers on either side with regard to the matter at issue in a
civil or criminal trial.
1. Basic Test
A lawyer who is participating or has participated in the investigation or litigation of a matter
must not make an extrajudicial statement that the lawyer knows or reasonably should know
PROFESSIONAL RESPONSIBILITY 45.
will be disseminated by means of public communication and will have a substantial likeli-
hood of materially prejudicing an adjudicative proceeding in the matter. This prohibition also
applies to any such lawyer’s associates in a firm or government agency. [RPC 3.6(a), (d)]
2. Permitted References
A lawyer may state:
(i) The claim, offense, or defense involved and, except when prohibited by law, the
identity of the persons involved;
(v) A request for assistance in obtaining evidence and information necessary thereto;
(vi) A warning of danger concerning the behavior of a person involved when there is
reason to believe that there exists the likelihood of substantial harm to an individual or
to the public interest; and
(vii) In a criminal case, in addition to the above, (i) the identity, residence, occupation, and
family status of the accused; (ii) if the accused has not been apprehended, information
necessary to aid in apprehension of that person; (iii) the fact, time, and place of arrest;
and (iv) the identity of investigating and arresting officers or agencies and the length of
the investigation.
[RPC 3.6(b)]
(v) Information the lawyer knows or reasonably should know is likely to be inadmissible
as evidence in a trial and would, if disclosed, create a substantial risk of prejudicing an
impartial trial; or
(vi) The fact that a defendant has been charged with a crime, unless there is included
therein a statement explaining that the charge is merely an accusation and that the
defendant is presumed innocent until and unless proved guilty.
Another relevant factor in determining prejudice is the nature of the proceeding. Thus,
criminal jury trials are most sensitive to extrajudicial speech, while civil trials may be less
sensitive. Nonjury hearings and arbitration proceedings may be even less affected. [See RPC
3.6, comments 5, 6]
4. Right of Reply
A lawyer is permitted to make a statement that a reasonable lawyer would believe is required
to protect a client from the substantial undue prejudicial effect of recent publicity not initi-
ated by the lawyer or her client. Any statement made pursuant to this rule must be limited to
information that is necessary to mitigate the recent adverse publicity. [RPC 3.6(c)]
A. IN GENERAL
Just as the Rules of Professional Conduct regulate the formation of lawyer-client relationships,
they also spell out the proper procedures for withdrawal from the representation of a client. The
Rules distinguish between situations where the lawyer must terminate employment (manda-
tory withdrawal) and situations where she is permitted to terminate employment (permissive
withdrawal).
B. MANDATORY WITHDRAWAL
Withdrawal is mandatory (after obtaining permission from the tribunal if required by its rules), or
representation should be declined if:
(i) The representation will result in violation of the Rules of Professional Conduct or other
law;
(ii) The lawyer’s physical or mental condition materially impairs the lawyer’s ability to repre-
sent the client; or
[RPC 1.16(a)]
C. PERMISSIVE WITHDRAWAL
A lawyer may withdraw from representing a client if:
(i) Withdrawal can be accomplished without material adverse effect on the interests of the
client;
PROFESSIONAL RESPONSIBILITY 47.
(ii) The client persists in a course of action involving the lawyer’s services that the lawyer
reasonably believes is criminal or fraudulent;
(iii) The client has used the lawyer’s services to perpetrate a crime or fraud;
(iv) A client insists upon taking action that the lawyer considers repugnant or with which the
lawyer has a fundamental disagreement;
(v) The client fails substantially to fulfill an obligation to the lawyer regarding the lawyer’s
services and has been given reasonable warning that the lawyer will withdraw unless the
obligation is fulfilled;
(vi) The representation will result in an unreasonable financial burden on the lawyer or has
been rendered unreasonably difficult by the client; or
[RPC 1.16(b)]
A. IN GENERAL
Given the unique nature of the legal profession in society, every lawyer bears the burden of
assisting in the improvement of the legal system and assuring that the profession itself is not
brought into disrepute. Many of the RPC provisions discussed so far attempt to recognize the
significance of this concept in the course of regulating relationships with a client and need not be
restated here. However, there are other provisions of the Rules, as well as some general principles,
that give further substance to the lawyer’s obligations to the legal system and profession. These
are set forth below.
his conduct by acting in a manner that promotes public confidence in the integrity and efficiency
of the legal system and profession.
1. Prohibited Contributions
This rule does not prohibit all political contributions by lawyers or firms—only those that
would not have been made but for the desire to be considered for the employment or
appointment. The circumstances of the contribution may indicate its purpose. Contributions
that are substantial compared to contributions made by other lawyers or firms, are made
for the benefit of an official who can award such work, and are followed by an award to the
lawyer or firm support an inference that the contributions were for the purpose of obtaining
the work. Other factors, such as a family or professional relationship with a candidate or a
desire to further a political, social, or economic interest, weigh against inferring a prohibited
purpose. [RPC 7.6, comment 5]
2. Excluded Employment
Excluded from the ambit of the rule are: (i) uncompensated services; (ii) engagements or
appointments made on the basis of experience, expertise, qualifications, and cost, following a
process that is free from influence based on political contributions; and (iii) engagements or
appointments made on a rotating basis from a list compiled without regard to political contri-
butions. [RPC 7.6, comment 3]
(i) Refrain from prosecuting a charge that the prosecutor knows is not supported by probable
cause;
(ii) Make reasonable efforts to ensure that the accused has been advised of the right to, and
the procedure for obtaining, counsel and has been given reasonable opportunity to obtain
counsel;
(iii) Not seek to obtain from an unrepresented accused a waiver of important pretrial rights such
as the right to a preliminary hearing;
(iv) Make timely disclosure to the defense of all evidence or information known to the prose-
cutor that tends to negate the guilt of the accused or mitigate the offense, and in connection
PROFESSIONAL RESPONSIBILITY 49.
with sentencing, disclose to the defense and to the tribunal all unprivileged mitigating infor-
mation known to the prosecutor, except when the prosecutor is relieved of this responsibility
by a protective order of the tribunal;
(v) Exercise reasonable care to prevent investigators, law enforcement personnel, employees, or
other persons assisting or associated with the prosecutor in a criminal case from making an
extrajudicial statement that the prosecutor would be prohibited from making under RPC 3.6
(relating to trial publicity) or this Rule;
(vi) Not subpoena a lawyer in a criminal proceeding to present evidence about a client unless
the prosecutor reasonably believes: (i) the information is not protected by privilege; (ii) the
evidence is essential to an ongoing investigation or prosecution; and (iii) there is no feasible
alternative to obtain the information;
(vii) Refrain from making extrajudicial comments that have a substantial likelihood of height-
ening public condemnation of the accused (except for statements that are necessary to inform
the public of the nature and extent of the prosecutor’s action and that serve a legitimate law
enforcement purpose);
(viii) Promptly disclose new, credible, and material evidence that creates a reasonable likelihood
that a convicted defendant did not commit the subject offense and, if the conviction occurred
in the prosecutor’s jurisdiction, make a reasonable investigation into the possible wrongful
conviction; and
(ix) Seek to remedy a conviction when the prosecutor knows of clear and convincing evidence
that a defendant in her jurisdiction was convicted for an offense that he did not commit.
[RPC 3.8]
1. Selection of Judges
The constitutions of most states specify how judges are to be selected. In some states, judges
are appointed by the governor or the state legislature, while in others they are elected by the
voters. In still other states, judges are initially appointed and later retained or rejected by the
50. PROFESSIONAL RESPONSIBILITY
voters. State judges can be removed from office or otherwise disciplined in accordance with
state constitutional and statutory provisions.
2) While driving under the influence of alcohol, Judge L ran a traffic signal
and violated other traffic laws. Judge L is subject to discipline.
3. Personal Relationships
A judge must not allow family, social, political, or other relationships to interfere with the
judge’s conduct or judgments. [CJC 2.4(B)]
Example: Judge A was assigned to hear a case in which a well-known legislator was
charged with a RICO violation. Judge A and the accused legislator are
members of the same charitable organization and the same political party.
PROFESSIONAL RESPONSIBILITY 51.
Judge A must not allow these relationships to influence her decisions in the
case.
3) When Judge D’s teenage daughter was charged with shoplifting, Judge D
called Judge E, to whom the daughter’s case was assigned. D said: “E, as a
fellow judge, I want to tell you that my little girl is a good kid who deserves a
break.” Judge D is subject to discipline.
a. Permissible Acts
The following acts are permissible, as long as the judge is sensitive to abuse of the
prestige of the judicial office [CJC 1.3, Comments]:
2) Judicial Selection
Judges can participate in the process of judicial selection by cooperating with
appointing authorities.
3) Character Witness
When properly summoned, a judge may testify as a character witness for someone.
However, judges must not appear voluntarily as character witnesses, and (except
where the demands of justice require) they should discourage people from
requiring them to serve as character witnesses.
Further, this category does not include an organization that is “dedicated to the preservation
of religious, ethnic, or cultural values of legitimate common interest to its members.” [CJC
3.6 and Comments]
Examples: 1) Judge G belongs to the Slovenian League, which limits its membership
to all descendants (regardless of sex or race) of persons from Slovenia, a
former republic of Yugoslavia. The object of the organization is to preserve
the culture and traditions of the Slovenian people. The organization does not
stigmatize as inferior those who do not fall within its membership require-
ments. Judge G may belong to the Slovenian League because it does not
practice “invidious discrimination.” [See Moser, The 1990 ABA Code of
Judicial Conduct: A Model for the Future, 4 Geo. J. Legal Ethics 731, 739-44
(1991)]
4) Judge K belongs to the Ashmount Golf and Tennis Club, which limits its
membership to 1,200 white male Protestants. The object of the organization
is to provide golf, tennis, and social facilities for its members. Conversations
in the clubhouse frequently concern business and professional matters, and
membership in the club offers significant business and professional advan-
tages. Judge K is subject to discipline. Note that Judge K would be subject
to discipline for using the Golf and Tennis Club (e.g., having lunch there as
a guest, renting a room for a meeting there, or occasionally using the sports
facilities as a guest) even if he were not a member.
6. Right to Be Heard
A judge must give every person who has a legal interest in a proceeding (or that person’s
lawyer) the right to be heard in accordance with the law. [CJC 2.6(A)]
7. Ex Parte Communications
“Ex parte” means one side only. An ex parte communication means a communication
between a judge and representative from one side of a matter when no representative from
54. PROFESSIONAL RESPONSIBILITY
the other side is present. A judge may not initiate, permit, or consider ex parte communica-
tions except in these three situations:
b. Mediation or Settlement
With the consent of the parties, the judge may confer separately with the parties and
their lawyers in an effort to settle or mediate a pending matter. [CJC 2.9(A)(4)]
(i) The judge reasonably believes that no party will gain a procedural, substantive,
or tactical advantage from the communication; and
(ii) The judge notifies the lawyers for the other parties of the essence of the communi-
cation and gives them an opportunity to respond.
[CJC 2.9(A)(1)]
a. Court Personnel
A judge may consult about a matter with court personnel whose function is to aid the
judge in carrying out adjudicative responsibilities (e.g., the judge’s law clerk) and with
other judges provided the judge does not abrogate the responsibility to personally decide
the matter. [CJC 2.9(A)(3)]
c. Other Communications
A judge must not have communications about a matter outside the presence of the
PROFESSIONAL RESPONSIBILITY 55.
parties’ lawyers with any person not mentioned above, unless the conditions stated in
7.c., supra, are satisfied. [CJC 2.9(A)(1)]
Example: In the middle of a trial, juror A telephoned Judge T at home and blurted
out that she had accidentally overheard a graphic radio report about
the trial and felt unable to continue as a juror. Judge T calmed A and
instructed her to come to court the next morning and report the incident
in the presence of the lawyers for the parties. Judge T handled the matter
properly.
(i) Respect the rights of the parties, but resolve issues without unnecessary expense or
delay;
(ii) Monitor cases closely to eliminate dilatory practices, avoidable delays, and unnecessary
costs;
(iii) Encourage settlements, but without forcing the parties to give up their right to adjudica-
tion;
(iv) Devote adequate time to judicial duties;
(v) Be punctual in attending court;
(vi) Be expeditious in deciding matters under submission; and
(vii) Insist that the parties, lawyers, and court personnel cooperate in achieving the objec-
tives stated above.
b. Judge as a Party
The duty to abstain from comment does not apply if the judge is a litigant in a personal
capacity. The duty does apply, however, if the judge is a litigant in an official capacity,
as in writ of mandamus proceedings. [CJC 2.10(D), and Comment (2)]
a. Judicial Misconduct
If Judge A receives information indicating a substantial likelihood that Judge B has
violated the CJC, Judge A must take “appropriate action.” What constitutes “appropriate
action” depends on the situation; it could range from simply speaking directly with
Judge B about the matter to reporting Judge B to the relevant disciplinary authority. If
Judge A has actual knowledge that Judge B has committed a violation of the CJC that
raises a substantial question about Judge B’s honesty, trustworthiness, or fitness for
office, then Judge A must report Judge B to the judicial disciplinary authority. [CJC
2.15(A), (C)]
b. Lawyer Misconduct
If a judge receives information indicating a substantial likelihood that a lawyer has
violated the RPC, the judge must take “appropriate action,” which means the same as
with respect to judicial misconduct (see supra). If a judge has actual knowledge that a
lawyer has committed a violation of the RPC that raises a substantial question about the
lawyer’s honesty, trustworthiness, or fitness to practice, then the judge must report the
lawyer to the relevant disciplinary authority. [CJC 2.15(B), (D)]
D. DISQUALIFICATION
a. Disclosure by Judge
The judge should disclose on the record any information the judge believes that the
parties or their lawyers might consider relevant to the question of disqualification, even
if the judge believes there is no reasonable basis for disqualification. [CJC 2.11 and
Comment (5)]
Example: Judge Y plans to retire from the bench at the end of the year and return
to private law practice. Judge Y has held tentative discussions with
the private firm of A, B & C about joining that firm. Now Judge Y is
assigned to hear a case in which the defendant is represented by the A, B
& C law firm. Judge Y should disclose the facts and let the parties decide
whether to waive disqualification.
b. Rule of Necessity
Case law has created a rule of necessity that overrides the rules of disqualification.
For example, suppose that Judge Z is the only judge available to rule on an emergency
motion for a temporary restraining order. Judge Z may rule on the motion even though
she might be disqualified were it not an emergency. Even in such a situation, Judge Z
should disclose the ground for disqualification on the record and should use reasonable
efforts to transfer the matter to a different judge as soon as possible. Further, a judge
should not be disqualified for a reason that would apply equally to all other judges to
whom the matter might be assigned.
Examples: 1) State trial judge A is assigned to hear a case concerning the consti-
tutionality of a statute that will raise the salary of all trial judges in the
state. Judge A may hear the case because the reason for disqualification
applies equally to all other judges to whom the case might be assigned.
personal and must stem from an extrajudicial source; adverse attitudes toward a party formed
on the basis of evidence presented in the case are not disqualifying.
3. Prior Involvement
A judge must disqualify himself if the judge previously:
(v) Was associated with a lawyer who substantially participated in the matter at the time
they were associated.
[CJC 2.11(A)(6)]
Examples: 1) Before her appointment as a state supreme court justice, Justice C practiced
law with lawyer L. At the time C and L were associated, L represented X in
the trial of X v. Y. After the trial, L withdrew as X’s lawyer. Now the case is
on appeal to the state supreme court. Justice C is disqualified.
3) Before taking the bench, Judge D was a lawyer for the United States
Justice Department. At that time, attorney A was also a lawyer for the Justice
Department. Now Judge D is assigned to hear a case in which A represents
the Justice Department. Ordinarily, a lawyer in a government agency is not
“associated” with the other lawyers in the agency for purposes of the CJC.
Thus, Judge D is not disqualified unless his prior work with A creates a
reasonable question about his impartiality under the general rule of disqualifi-
cation (see D.1., supra).
4. Economic Interest
A judge must disqualify himself if the judge knows that he, either as an individual or as a
fiduciary, has an economic interest in the matter or in one of the parties. [CJC 2.11(A)(3)]
Disqualification is also required if the interest is held by the judge’s spouse, domestic partner,
parent, or child (wherever residing) or by any other member of the judge’s family who resides
in the judge’s household. A judge must keep informed about his economic interests, and must
make a reasonable effort to keep informed about those of the judge’s spouse or domestic
partner and minor children residing in the judge’s household. [CJC 2.11(B)]
60. PROFESSIONAL RESPONSIBILITY
(i) Is an officer, director, adviser, or other active participant in the affairs of a party;
or
(iii) Having a deposit at a bank, credit union, or savings association that is a party
(unless the proceedings could substantially affect the value of the deposit); or
(iv) Owning government securities and the government is a party (unless the value of
the securities could be substantially affected by the proceedings).
5. Involvement of a Relative
A judge must disqualify himself if the judge has a relative involved in the case. [See CJC
2.11(A)(2)]
a. Meaning of “Relative”
For the purpose of this rule, the term “relative” means a person (or the spouse or
domestic partner of a person) who is related within the third degree to the judge or to
the judge’s spouse or domestic partner. The third degree of relationship means: great-
grandparents, grandparents, parents, uncles, aunts, brothers, sisters, children, grand-
children, and great-grandchildren; in short, anyone related more closely than a cousin.
Remember that spouses and domestic partners are included on both ends of the calcula-
tion.
Example: Judge H is married to Mable. The third husband of Mable’s Aunt Lulu is
the plaintiff in a case assigned to Judge H. Judge H is disqualified.
b. Meaning of “Involved”
The term “involved” means that the relative is:
(iii) Known by the judge to have more than a de minimis interest that could be substan-
tially affected by the proceedings; or
Example: Judge J is assigned to hear a case in which the state Attorney General
seeks suspension of the license of the Shady Acres Nursing Home until
Shady Acres provides more humane living conditions for its residents.
Judge J’s husband’s great-grandmother is a Shady Acres resident.
Because the great-grandmother’s interests could be substantially
affected, Judge J is disqualified.
8. Remittal of Disqualification
The parties and their lawyers can remit (waive) all of the foregoing grounds of disqualifica-
tion, except personal bias or prejudice concerning a party. [CJC 2.11(C)] The procedure for
remittal is as follows:
(i) The judge discloses on the record the ground for disqualification. The judge may then
ask whether the parties and their lawyers wish to discuss waiver.
(iii) All the parties and their lawyers meet, outside the presence of the judge, and agree that
the judge should not be disqualified. As a practical matter, the judge may wish to have
all the parties and their lawyers sign a remittal agreement.
(iv) If the judge is willing to do so, he may then proceed with the case.
E. EXTRAJUDICIAL ACTIVITIES
A judge may engage in extrajudicial activities but must not:
(i) Participate in activities that undermine the judge’s independence, integrity, or impartiality;
(iii) Participate in activities that interfere with the judge’s judicial duties or lead to frequent
disqualification; or
[CJC 3.1]
1. Avocational Activities
A judge may speak, write, lecture, teach, and participate in nonjudicial activities that involve
either legal or nonlegal subjects, provided that these activities are consistent with the duties
stated elsewhere in the CJC. Because judges are in a unique position to help improve the law,
they are encouraged to do so through bar associations, judicial conferences, and the like.
[CJC 3.1, Comments (1), (2)]
2) Judge N met privately with the Mayor of the city of Glenview to protest the
city’s plan to open a city dump adjacent to Judge N’s property. The meeting
was proper because it concerned Judge N’s own interests.
(i) Will be engaged in proceedings that would ordinarily come before the judge; or
(ii) Will frequently be engaged in adversary proceedings in the court on which the
judge sits or one under its appellate jurisdiction.
[CJC 3.7(A)(6)]
2) Judge S is the only trial judge who sits in Oceanside County. She is
invited to be a trustee of the Oceanside Memorial Hospital, which is
frequently named as a defendant in medical malpractice cases filed in
Oceanside County. Judge S must decline the invitation.
(vii) Commercial and financial opportunities, including loans from a lending institution,
made in the regular course of its business and on the same terms available to nonjudges;
and
(viii) A scholarship or fellowship or similar benefit given on the same terms as to other
people.
[CJC 3.13]
(ii) An invitation to the judge and his spouse, domestic partner, or guest to attend a
law-related function or an event associated with the judge’s educational, religious,
charitable, fraternal, or civic activities; and
(iii) Gifts, bequests, loans, or other benefits from a person who has come or is likely to
come before the judge.
[CJC 3.13(C)]
7. Fiduciary Activities
Generally, a judge may not serve as an executor, administrator, trustee, guardian, or other
fiduciary. However, a judge may serve in such a capacity for a member of the judge’s family,
but only if the service will not:
(ii) Involve the judge in proceedings that would ordinarily come before him; or
(iii) Involve the judge in adversary proceedings in the court on which the judge sits or one
under its appellate jurisdiction.
b. Conflicting Duties
When the duties of a fiduciary conflict with the judge’s duties under the CJC, the judge
should resign as fiduciary. [CJC 3.8, Comment (1)]
Example: Judge V is appointed as trustee of a fund for the use and benefit of
his invalid brother. The trust fund includes common stock of several
companies that frequently appear as litigants before Judge V. The CJC
requires a judge to manage her investments in a way that minimizes
66. PROFESSIONAL RESPONSIBILITY
(i) The compensation or expense reimbursement does not undermine the judge’s
independence, integrity, or impartiality or influence the judge’s performance of
judicial duties;
(ii) The compensation is reasonable for the work done and does not exceed what would
be given to a nonjudge; and
(iii) The reimbursement of expenses does not exceed actual expenses reasonably
incurred by the judge and, when appropriate to the occasion, the judge’s spouse,
domestic partner, or guest.
[CJC 3.12; 3.14]
b. Reports of Compensation
A judge who receives compensation or reimbursement for outside activities must report:
the activity, when and where it took place, the payor, and the amount. A judge must
make such reports annually in a public court document, but reports for reimbursement
must be made within 30 days of the event. In a community property state, compensation
received by the judge’s spouse is not attributed to the judge for this purpose. [CJC 3.15]
a. Definition of “Candidate”
A “candidate” is a person who seeks to obtain or retain a judicial office either by
election or appointment. The same definition applies to a judge who seeks an elected or
appointed nonjudicial office. A person becomes a candidate when she does any one of
the following things:
b. General Prohibitions
Except where specifically permitted by the CJC, a judge or candidate for judicial office
must not:
(v) Publicly identify herself as a candidate of a political organization and seek, accept,
or use endorsements from a political organization;
(vii) Personally accept or solicit campaign contributions other than through a campaign
committee, or use campaign contributions for private use.
[CJC 4.1] These general prohibitions have some exceptions that are discussed in 3.,
infra.
(i) Communicate with the appointing authority and screening groups; and
(ii) Seek support from any person or organization other than a political partisan organiza-
tion.
[CJC 4.3]
[See CJC, Terminology (definition of candidate)] These three classes of persons may do the
following things:
(ii) Speak on behalf of her candidacy, including through advertisements, websites, or other
campaign literature;
(iii) Publicly endorse or oppose candidates for the same judicial office;
(iv) Seek, accept, or use endorsements from a person or organization other than a partisan
political organization;
[CJC 4.2]
In addition to the above, candidates in partisan public elections may identify themselves as
candidates of a political organization and seek, accept, and use endorsements of a political
organization. [CJC 4.2]
a. Campaign Activities
These parties may engage in campaign activities, subject to the following rules:
70. PROFESSIONAL RESPONSIBILITY
2) Campaign Committees
These parties may, however, establish campaign committees, which may: (i) put
on candidate forums and publish campaign literature; (ii) manage campaign funds;
and (iii) solicit public support and reasonable contributions from members of the
public, including lawyers. Note that if a judge knows the identity of contributing
lawyers or litigants, that fact may require the judge to disqualify herself under the
CJC (see D., supra). [CJC 4.4 and Comments]
1. A retired judge subject to recall is allowed to serve as an arbitrator or mediator, and (except
when acting as a judge) to serve as a fiduciary.
2. Continuing part-time judges, periodic part-time judges, and pro tempore part-time judges
are exempt from many, but not all, of the CJC provisions that restrict outside activities and
political activities.
PROFESSIONAL RESPONSIBILITY MULTIPLE CHOICE QUESTIONS 1.
INTRODUCTORY NOTE
You can use the sample multiple choice questions below to review the law and practice your under-
standing of important concepts that you will likely see on your law school exam. To do more questions,
access StudySmart MPRE software from the BARBRI website.
Question 1 Question 2
The mayor of a small city is also a licensed A sales manager testified before a federal
attorney who has a law partnership with a fellow grand jury that was investigating price-fixing
attorney in the city. Under the city’s charter, in the automobile tire industry. Ultimately, the
the mayor has the authority to determine what grand jury indicted the sales manager for price-
issues are to be placed upon the agenda of the fixing, a felony under the Sherman Act. After
city council. Several council members have told his indictment, the sales manager sought to hire
the mayor that they would like to see a particular a prominent attorney to represent him at his
zoning measure placed upon the agenda. This criminal trial. The sales manager is a middle
proposed ordinance would ban commercial class business executive with enough savings
development of a certain area within the city to pay for private counsel. He told his attorney
limits. in confidence that he had lied to the grand jury
about several meetings he had had with competi-
The mayor’s law partner has been retained tors. Furthermore, he told her that he wanted to
as attorney for a development company that has plead not guilty to the criminal charge and that
acquired land in the proposed noncommercial he intended to testify at trial as he did before the
zoning area and has plans to construct a large grand jury.
shopping center there. The mayor has agreed to
take no direct role in the representation, not to Which of the following would be proper for
share any fees from the case, and not to attend the attorney to do in this situation?
any city council meetings at which the matter
will be discussed. (A) Decline to represent the sales manager.
May the mayor’s law partner represent the (B) Agree to represent the sales manager and
development company in this matter? tell no one what he told her.
(A) Yes, because the development company is (C) Inform the sales manager that unless he
not a client of the mayor. pleads guilty to the criminal charge, she
will tell the prosecutor about his false testi-
(B) Yes, because the mayor will not be present mony before the grand jury.
at any city council meetings at which the
matter is discussed. (D) Decline to represent the sales manager and
inform the prosecutor about his false testi-
(C) No, because of the mayor’s position as mony before the grand jury.
mayor.
Question 3 Question 4
A partner in a law firm has just been elected a A law school graduate who is not a licensed
judge of the circuit court. She has been assigned member of the bar felt called to the ministry
to the probate division. During her last week after he graduated from law school. He received
with the law firm, she filed a number of very a degree in divinity and was formally ordained
routine, uncontested probate motions. At the as a minister of his faith. He is now the pastor of
time, she had no idea that she would be assigned a local church, where an attorney is a member
to the probate division. These routine probate of the congregation. As pastor, he has been very
motions have been assigned to her courtroom by disturbed about the high rates of divorce and the
a lottery system of random assignment that the breakdown in American family life. Therefore,
circuit court regularly employs to assign cases. he holds frequent “family counseling sessions”
where, among other things, he explains to the
Is it proper for the judge to rule on these parishioners who attend these sessions many
motions? of the legal ramifications of divorce, alimony,
child support, and child custody. These sessions
(A) Yes, because they are routine and uncon- are usually followed by question-and-answer
tested. periods, during which the pastor gives legal
advice to parishioners who cannot afford a
(B) Yes, because reassignment would cause lawyer. The pastor knows that the legislature
delay. has passed a new marriage dissolution law that
changes the law substantially from what he
(C) No, because she has a conflict of interest. was taught in law school. The pastor asks the
attorney who is a member of the congregation if
(D) No, because judges may never rule on he will prepare an outline and a memorandum
issues when their former law firm is fully explaining the new law so that he will be
involved. better informed for the sessions with his parish-
ioners.
Question 5 Question 6
After a major airplane crash in the vicinity A lawyer represents a defendant who is being
of an affluent island town, in which 122 passen- prosecuted in a jury trial for an armed robbery
gers and crew were killed, the town’s attorneys and attempted murder that occurred on June
swarmed like locusts to get a “piece of the 15. The accused has pleaded not guilty to the
action” and the potentially huge contingent fees charges, but the lawyer knows that the accused
that were likely to arise from the case. Inter- is the perpetrator and that the crime occurred
ested in fees himself, but also rather disgusted at approximately 10 p.m. The victim testifies
at the performance of some of his colleagues of that she is certain that the crime occurred at
the bar, one attorney placed an ad in the town’s midnight. The accused has an airtight alibi for
weekly legal newspaper, whose readership midnight. At 11:40 p.m. he was arrested on
was almost entirely lawyers. The ad suggested a drunk driving charge, and he was in police
that any attorneys representing plaintiffs in custody until 6 a.m. on June 16. On cross-exami-
the airline crash matter contact him in order nation, the lawyer does nothing to challenge the
to consolidate lawsuits against the airline, and victim’s recollection of the time of the attack.
that legal fees would be divided in proportion Also, as the trial unfolds, the lawyer does not
to the work performed. The ad was signed by introduce any evidence at her disposal that
the attorney and indicated his office address and would help establish the time of the attack as
telephone number. 10 p.m. The lawyer calls as a witness a police
officer who testifies that the accused was in fact
Was it proper for the attorney to place such an in custody at midnight on the night in question.
advertisement? The accused does not testify and is acquitted.
(A) Yes, because the ad was not misleading. Are the lawyer’s actions proper?
(B) Yes, because the lawyers will split the fees (A) Yes, because her client is a criminal de-
in proportion to work done. fendant and constitutional protections take
precedence over ethical rules.
(C) No, because the attorney is soliciting
business. (B) Yes, because she did not present false
evidence.
(D) No, because the ad is in bad taste.
(C) No, because she knew that the victim’s
testimony was wrong and would mislead
the jury as to a crucial component of the
case.
Question 7 Question 8
An attorney has a high-profile divorce law A man walked into the law offices of a lawyer
practice in the town in which he resides. Because who, because she was not busy at the time,
of his heavy caseload, the attorney often appears agreed to talk to the man right away. The man
before the four chancery judges of the county told the lawyer that he was concerned that he
court. One of the chancery judges is getting might be indicted soon. He explained the details
married, and he sends a wedding invitation to of his predicament at length to the lawyer, but
the attorney. The attorney wishes to send the after he finished, the lawyer explained that she
judge, as a wedding gift, an imported Italian only handled civil matters and urged the man to
machine that makes espresso and cappuccino consult with a friend who was a criminal lawyer.
coffee because he knows that the judge loves The man went on to retain the friend. A few days
fine coffee. The coffee machine sells for $200 after her interview with the man, the lawyer read
at the town’s best cooking equipment store. The a news item announcing the man’s indictment. It
attorney sent the coffee machine to the judge, quoted the district attorney at some length. After
and the judge duly made a public report of the reading the article, the lawyer became convinced
gift. that something the man had told her during their
interview would probably exonerate him or, at
Was it proper for the attorney to send the the very least, lead to a reduction in the charges
coffee machine to the judge? against him if the district attorney became aware
of the information in her possession.
(A) Yes, because the judge would not be un-
duly influenced by a $200 gift. May the lawyer reveal the information to the
district attorney?
(B) Yes, because the judge made a public report
of the gift. (A) Yes, because the man did not retain the
lawyer as his counsel.
(C) No, because the gift was not a campaign
contribution, and lawyers should not give (B) Yes, because the information will help the
other types of gifts to judges. man.
(D) No, because the value of the gift exceeded (C) No, because the attorney learned the infor-
$150. mation during the course of an attorney-
client relationship.
Question 9 Question 10
A lawyer practices in State A. State A’s rules A wife whose husband was hard-drinking
of legal ethics depart from the ABA Model and abusive went to see a lawyer about getting
Rules in one significant respect: State A has no a divorce. However, because the wife did not
“financial injury” exception to the lawyer’s duty work outside the home, she told the lawyer that
of confidentiality. Thus, when a State A lawyer she could not afford to pay a big legal fee. The
learns in confidence that her client is about to lawyer was sure that the wife had adequate
use her legal services to inflict serious financial grounds for divorce in that her husband was
injury on someone, the lawyer may withdraw, adulterous, an alcoholic, and frequently beat
but she must not reveal what she learned in her. Therefore, the lawyer told the wife that if
confidence. The lawyer limits her practice to she could put up the $200 filing fee, he would
federal securities law, and she regularly appears do all the work for 10% of whatever he was able
before the Securities and Exchange Commission to obtain in alimony and child support on her
(“SEC”). One of her major clients is a company behalf. The wife was elated and immediately
that makes and sells cotton textiles. The compa- agreed to the arrangement, thanking the lawyer
ny’s shares are traded on the New York Stock profusely. The lawyer left his office that night
Exchange and in securities matters the company with a warm feeling that he had helped a fellow
is regulated by the SEC. While working on an human being, secure in the knowledge that at
SEC registration statement for the company, least one person in the community would have
the lawyer learned in confidence that three of something nice to say about lawyers.
the company’s top executives were cooperating
in a scheme to loot the company of millions of Personal satisfaction aside, was the lawyer’s
dollars. If their scheme continues, it could drive conduct proper?
the company into insolvency. The lawyer alerted
the chief legal officer of the company to the (A) Yes, because the fee is reasonable and is
situation, but he did nothing. She then alerted not excessive.
the chief executive officer, who also did nothing.
Finally, she alerted the six outside members of (B) Yes, because the lawyer is providing legal
the board of directors, but they too failed to act. services to a person who might not other-
In disgust, the lawyer withdrew from the matter wise be able to afford them.
and vowed never again to represent the company.
(C) No, because a contingent fee arrangement
Must the lawyer now tell the SEC about the is prohibited when the fee is based on the
scheme? amount of alimony or a division of marital
property.
(A) Yes, the SEC’s regulations under the Sar-
banes-Oxley Act require her to alert the SEC (D) No, because the wife was indigent and the
if her other efforts have proven fruitless. lawyer should have advanced her the filing
fees.
(B) No, the SEC’s regulations give her discre-
tion to either reveal or not reveal the matter
to the SEC.
(C) Yes, because the shareholders could be
seriously financially injured if the scheme
continues.
(D) No, because State A’s legal ethics rules do
not allow her to reveal confidential informa-
tion in this situation.
6. PROFESSIONAL RESPONSIBILITY MULTIPLE CHOICE QUESTIONS
Question 11 Question 12
The driver of a car and his passenger were A judge ruled in favor of a plaintiff in a civil
injured as the result of a collision with a bus. action where the defendant was ordered to pay
They believe the bus driver was entirely at fault, the plaintiff $50,000 in damages. The judge has
and they want to bring a negligence action since resigned from the bench. The defendant
against the bus company. They engage in an has refused to pay the $50,000, asserting that the
initial consultation with a local attorney. In the verdict was obtained through improper means.
course of the consultation, the attorney realizes The defendant asks the judge, now in private
that the bus at issue belongs to a bus company practice, if she will represent him.
that the attorney’s firm is representing in an
unrelated matter. The attorney interrupts the Would the judge be subject to discipline if she
conversation, explains this potential conflict of represents the defendant?
interest, and obtains the written consent of both
the driver and passenger to represent them. (A) No, because the judge is no longer on the
bench.
Which of the following is not true?
(B) No, because the judge was not a party to
(A) The attorney may not represent the car’s fraud when the original verdict was handed
driver and passenger because he has not down.
obtained the written consent of the bus
company. (C) Yes, because the judge ruled on this case
when she was a judge.
(B) The attorney may not represent the car’s
driver and passenger because he did not (D) Yes, because former judges may not engage
inform the passenger that he may have a in private practice.
cause of action against the car driver.
Answer to Question 1
(C) If one lawyer within a firm has a conflict of interest and cannot take on a matter, no other lawyer
in the firm may take on the matter either. [ABA Model Rule 1.10(a)] One situation that would
create a concurrent conflict would be if there is a significant risk that the representation of a client
will be materially limited by the lawyer’s own interest or his responsibilities to another client,
a former client, or a third person. [ABA Model Rule 1.7(a)(2)] The mayor would be prohibited
from representing the development company in this matter because such representation would
be materially limited by his responsibilities, as mayor, to the city. Thus, his law partner is also
prohibited from such representation. Therefore, (A) and (B) are incorrect. (D) is also incorrect; it
makes no difference whether the mayor has a direct role in the representation or shares any fees.
Answer to Question 2
(A) The attorney has no duty to represent the sales manager, so (A) is proper. (B) is improper because
the attorney may only represent the sales manager if he does not insist on testifying falsely. [See
ABA Model Rule 3.3(a)(3)] (C) is improper because it is a form of extortion. (D) is improper
because the sales manager’s confession to past perjury is protected by the duty of confidentiality.
[See ABA Model Rule 1.6]
Answer to Question 3
(C) A judge should disqualify herself in a proceeding in which her impartiality might reasonably be
questioned, including but not limited to instances where she served as lawyer in the matter in
controversy. [CJC Rule 2.11(A)(6)(a)] The fact that these matters are routine or uncontested does
not excuse her from this rule; thus (A) is incorrect. (B) is incorrect because avoiding delay does
not allow a judge to ignore CJC Rule 2.11(A)(6)(a). (D) is incorrect because it is too broad.
Answer to Question 4
(B) Under ABA Model Rule 5.5(b), a lawyer must not aid a nonlawyer in the unauthorized practice of
law. Under these facts, the attorney would improperly be assisting the pastor, who has not in fact
been authorized to practice law despite his law school degree. (A) makes no sense; the pastor’s
potential remedy for the attorney’s mistakes does not subject the attorney to discipline. (C) is
incorrect because any duty a lawyer may have to educate the public does not justify a lawyer’s
assisting in the unauthorized practice of law. (D) is irrelevant.
Answer to Question 5
(A) This is the best answer here because nothing in this advertisement violates the ABA Model Rules.
The fact that the ad is not misleading is important because neither the ABA Model Rules nor
the First Amendment protects misleading or deceptive advertising. (B) is incorrect because fee
splitting is a separate issue; it does not affect the propriety of the ad. (C) is incorrect because the
attorney here is not soliciting business, and also because the traditional ban on all solicitation
is no longer constitutional. (D) is incorrect in that the ad is not patently in bad taste, and even
if it were, it would probably be protected by the First Amendment unless it was misleading or
overreaching.
8. ANSWERS TO MULTIPLE CHOICE QUESTIONS
Answer to Question 6
(B) The lawyer’s actions were proper because she did not offer false evidence, and she is under no
duty to volunteer harmful facts. [See ABA Model Rule 3.3; comment 14 to ABA Model Rule 3.3]
In fact, to do so would probably be a breach of ethics. (A) is incorrect because her actions were
proper regardless of the constitutional protections afforded criminal defendants. (C) is incorrect
because an attorney is under no obligation to volunteer harmful facts in an adversarial proceeding
even if the jury will be misled by the testimony of a witness. It is up to the state to establish the
time of the crime; if it cannot do so, it has not met its burden of proof. (D) is incorrect because
the lawyer should not disclose the facts to anyone, not even the judge. These facts are informa-
tion related to the case and cannot be disclosed or used to the client’s disadvantage absent some
recognized exception to the duty of confidentiality. None applies here. Had the lawyer presented a
witness (other than the accused) who testified that the time was midnight when the lawyer knew
it was 10 p.m., the lawyer would have had to rectify the false testimony. Here, however, the testi-
mony came from the opponent, and the lawyer is under no obligation to rectify it.
Answer to Question 7
(B) ABA Model Rule 3.5(a) forbids a lawyer from seeking to influence a judge by means prohibited
by law. CJC Rule 3.13(C)(3) permits a judge to accept a gift from someone who has come or is
likely to come before the judge if the judge files a public report of the gift. (A) is wrong because
there is no specific value that is tied to undue influence. (C) is wrong because campaign contribu-
tions are not the only types of gifts lawyers may make to judges. (D) is wrong because the value
of the gift triggers only a reporting requirement. The gift is not improper as such.
Answer to Question 8
(D) An attorney may reveal or use confidential information if the client gives informed consent. [See
ABA Model Rules 1.6, 1.18] (C) is incorrect because it does not take into account exceptions to
the general rule of confidentiality. (A) is incorrect because the man consulted the lawyer in her
capacity as an attorney, and it is irrelevant that he did not retain her. (B) is incorrect because the
attorney may not reveal information, even if it will help the client, unless the client consents or the
information falls into recognized exceptions to the confidentiality rule, which are not present here.
Answer to Question 9
(B) The SEC’s regulations under the Sarbanes-Oxley Act permit, but do not require, a securities
lawyer to reveal a client’s confidential information to the SEC when the lawyer reasonably thinks
that doing so is necessary to prevent or rectify a securities act violation (or similar law violation)
that is likely to cause substantial financial injury to the client or its shareholders. (A) and (C) are
wrong because the regulation permits, but does not require, the lawyer to alert the SEC. (D) is
wrong because the SEC regulations purport to preempt any inconsistent state ethics rules. [See
17 C.F.R. §§205.1, 205.6(c)] (It remains to be seen whether courts will uphold the SEC’s effort to
preempt the field and override inconsistent state ethics rules.)
Answer to Question 10
(C) ABA Model Rule 1.5(d)(1) subjects a lawyer to discipline if the fee in a domestic relations matter
is contingent upon the securing of a divorce, the amount of alimony or support, or the amount
of the property settlement. (A) is incorrect because the size of the fee is irrelevant because the
ANSWERS TO MULTIPLE CHOICE QUESTIONS 9.
ABA Model Rules forbid a contingency fee in this situation no matter what the amount of the
fee. Although (B) states a rationale behind contingency fees, this rationale will not support a
contingency fee in this case. (D) is incorrect because a lawyer may, but is not required to, pay or
advance costs to an indigent client.
Answer to Question 11
(C) The attorney may not represent both the passenger and the driver when there is a potential
conflict of interest between them unless: (i) the attorney reasonably believes that he can repre-
sent both clients effectively; and (ii) the passenger and the driver give informed, written consent.
The attorney must withdraw from the joint representation, however, if later discovery shows that
the passenger has an actual claim against the driver. Here, there is no indication that the driver,
the passenger, and the bus company all gave informed, written consent and thus (C) is not a true
statement. [See ABA Model Rule 1.7] (A) is a true statement for the same reason. (B) is true
because a lawyer must not represent a client if the representation of that client will be directly
adverse to the representation of another client, unless both clients give informed, written consent.
[See ABA Model Rule 1.7] (D) is true because the attorney already represents the bus company in
the unrelated matter.
Answer to Question 12
(C) ABA Model Rule 1.12(a) provides that a lawyer must not represent anyone in connection with a
matter in which the lawyer participated personally and substantially as a judge, unless all parties
to the proceedings give informed, written consent. Thus, (C) is correct, and (A) and (B) are incor-
rect. (D) is incorrect because a judge may subsequently engage in private practice, except she may
not participate in cases where she was personally and substantially involved.
APPROACH TO PROFESSIONAL RESPONSIBILITY 1.
APPROACH TO EXAMS
PROFESSIONAL RESPONSIBILITY
IN A NUTSHELL: Lawyers and judges are subject to various sources of ethical regulation. Most
importantly, the American Bar Association has adopted the Rules of Professional Conduct (“RPC”)
and the Code of Judicial Conduct (“CJC”), which outline the rights and duties of lawyers and judges.
Most states have rules that are patterned after the RPC and CJC. Lawyers and judges who violate
these rules may be subject to discipline.
A. Admission to Practice
1. Citizenship or residency requirements impermissible
2. Applicant must show good moral character
3. Lawyer cannot make false statements regarding applications
B. Disciplinary Process
1. General substantive standards
a. Violating the RPC or assisting another in doing so
b. Committing a criminal act that reflects adversely on lawyer’s honesty,
trustworthiness, or fitness as a lawyer in other respects
c. Stating or implying an ability to improperly influence a government
agency or official or to achieve results by means that violate the RPC or
other law
d. Conduct involving dishonesty, fraud, deceit, or misrepresentation
e. Conduct prejudicial to the administration of justice
f. Knowingly assisting a judge or judicial officer in conduct that violates
the CJC or other law
2. Common types of discipline: disbarment, suspension, and censure
3. Practice in multiple states
a. Lawyer is subject to the regulation of the state in which she is admitted,
regardless of where her conduct occurs
b. Choice of law
1) If conduct is related to the matter before the tribunal, tribunal
will apply the local RPC
2) For other conduct, tribunal will apply:
a) The RPC in state where conduct occurred, or
b) The rules of another state in which the predominant
effect of the conduct occurred
(1) Safe harbor for lawyer who reasonably believed
the predominant effect occurred elsewhere
A. Forms of Practice
1. May not practice under a trade name or other misleading name
2. May not partner with or share legal fees with nonlawyers
A. Advertising
1. Ads must not be false or misleading
a. Characteristics of misleading ads
1) Unfounded conclusions
2) Unjustified expectations of results
3) Unsubstantiated comparisons to other lawyers
2. Ad must contain the name and office address of at least one lawyer or firm
responsible for its content
APPROACH TO PROFESSIONAL RESPONSIBILITY 3.
A. Duty as Adviser
1. Exercise independent judgment and render candid advice
B. Scope of Representation
1. Abide by client’s decisions (e.g., accepting settlement offers)
2. Act within bounds of the law
APPROACH TO PROFESSIONAL RESPONSIBILITY 7.
D. Conduct of Litigation
1. Must bring only meritorious claims and contentions
2. Must make reasonable efforts to expedite litigation
G. Trial Publicity
1. Must not make extrajudicial statements likely to materially prejudice an adjudi-
cative proceeding
A. Declining Employment
1. Should not accept representation if it:
a. Will harass or maliciously injure another
b. Presents an unwarranted claim or defense
8. APPROACH TO PROFESSIONAL RESPONSIBILITY
B. Mandatory Withdrawal
1. Continued employment will violate the RPC
2. Lawyer’s physical or mental condition materially impairs ability to represent
client
3. Client discharges lawyer
C. Permissive Withdrawal
1. No material adverse effect on client’s interests
2. Client persists in a criminal or fraudulent course of action
3. Client used lawyer’s services to perpetrate a crime or fraud
4. Lawyer considers client’s course of action repugnant
5. Client fails to fulfill obligation to lawyer and has been reasonably warned that
lawyer will withdraw unless it is fulfilled
6. Representation will impose unreasonable financial burden
7. Other good cause
1) Permitted actions
a) Recommendations or references based on personal
knowledge
b) Participating in judicial selection
c) Testifying as character witness when summoned (must
not volunteer)
d. Must not use, or be a member of, an organization that practices invid-
ious discrimination
1) Exceptions: intimate, purely private organizations and organi-
zations that preserve religious, ethnic, or cultural values of
common interest to the members
e. Must not publicly manifest a knowing approval of invidious discrimina-
tion on any basis
15. Must require court staff and others under judge’s direction to act in accordance
with the CJC
C. Disqualification
1. Whenever judge’s impartiality might reasonably be questioned
a. Should disclose any information that parties and lawyers might consider
relevant, even if judge believes there is no basis for disqualification
2. Bias or personal knowledge of relevant evidentiary facts
3. Prior involvement in the matter
4. Sufficient economic interest in the matter
5. Relative involved in the case (related more closely than a cousin)
6. Party or party’s lawyer made contributions to judge’s campaign
fund over a specified amount
7. Prior public statement of judicial commitment to an issue in the proceeding
8. All of the above grounds for disqualification may be remitted (waived) by the
parties except personal bias or prejudice concerning a party
D. Extrajudicial Activities
1. May participate in nonjudicial activities that involve either legal or nonlegal
subjects if consistent with CJC
2. Must not appear at a public hearing before, or otherwise consult with, an execu-
tive or legislative body or official, except on matters concerning the law, the
legal system, or the administration of justice
3. Must not accept appointments to governmental committees or commissions
unless they concern the law, the legal system, or the administration of justice
4. May participate in activities sponsored by law-related organizations and
nonprofit organizations
5. May hold and manage personal or family investments
6. May not serve on the board or as an employee or adviser of a corporate entity
7. Should not accept gifts, bequests, favors, loans, or other things of value from
anyone if doing so violates the law or compromises judge’s independence,
integrity, or impartiality
a. Exceptions
1) Items of little intrinsic value
2) Prizes from random drawings open to nonjudges
3) Gifts incident to the activities of a family member that inciden-
tally benefit the judge
4) Ordinary social hospitality
5) Gifts from family and friends
b. Gifts subject to reporting requirements
1) Gifts incident to a public testimonial
2) Invitation to a law-related function associated with judge’s
activities
3) Gifts from a person who has or is likely to come before the judge
8. Judge may serve as an executor, guardian, etc., for family members only and in
limited circumstances
9. A full-time judge may not act as a mediator, arbitrator, or private judge unless
expressly authorized by law
APPROACH TO PROFESSIONAL RESPONSIBILITY 11.
The essay questions that follow have been selected to provide you with an opportunity to experience
how the substantive law you have been reviewing may be tested in the hypothetical essay examination
question context. These sample essay questions are a valuable self-diagnostic tool designed to enable
you to enhance your issue-spotting ability and practice your exam writing skills.
It is suggested that you approach each question as though under actual examination conditions. The
time allowed for each question is 45 minutes. You should spend 10-15 minutes spotting issues, under-
lining key facts and phrases, jotting notes in the margins, and outlining your answer. If you organize
your thoughts well, about 30 minutes will be more than adequate for writing them down. Should you
prefer to forgo the actual writing involved on these questions, be sure to give yourself no more time for
issue-spotting than you would on the actual examination.
The BARBRI technique for writing a well-organized essay answer is to (i) spot the issues in a
question and then (ii) analyze and discuss each issue using the “CIRAC” method:
C — State your conclusion first. (In other words, you must think through your answer before you
start writing.)
I — State the issue involved.
R — Give the rule(s) of law involved.
A — Apply the rule(s) of law to the facts.
C — Finally, restate your conclusion.
After completing (or outlining) your own analysis of each question, compare it with the BARBRI
model answer provided herein. A passing answer does not have to match the model one, but it should
cover most of the issues presented and the law discussed and should apply the law to the facts of the
question. Use of the CIRAC method results in the best answer you can write.
2. PROFESSIONAL RESPONSIBILITY EXAM QUESTIONS
A third year law student went to interview for a job with Sam Stern, a local lawyer. The sign on the
door read:
Inside was a scene of chaos: telephones ringing, files stacked around randomly, and clients waiting.
As the student sat in the waiting room, he could tell that the receptionist was discussing the scandalous
details of one of the divorce cases in the office with a caller who had reached her on her personal cell
phone. Finally lawyer Stern arrived. As he breezed by on the way to his inner office, the receptionist
said, “Mrs. Barnes is holding for you on two. She said she hasn’t been able to talk to you about her
personal injury case for months.” The lawyer paused, seemed to think about that for a second and said,
“Take a number and tell her I’ll get back to her this afternoon.” The student watched as the receptionist
wrote up a message and placed it on an old-fashioned spindle already overflowing with messages.
Apparently reminded that he had a law student waiting for an interview, Sam Stern burst back out
of his office and said, “Come with me, I’m late for a deposition. We’ll talk in the car.” He paused
and asked his receptionist, “Can I have a check on the office account so I can take this young man to
lunch?” When told that the balance was too low, he responded, “There’s plenty of money in the trust
account, give me one of those checks—I’ll pay that account back when the Jones fee comes in.”
His receptionist paused, started to say something, but then handed him a check on his trust account.
“One other thing before you leave: Judge Brown is holding on line one. He wants to appoint you to
serve as guardian ad litem in a divorce case.” As he went through the door, Mr. Stern yelled, “I’m not
going to take any of those low-paying appointments anymore. Tell him I’ve got a conflict—I’ll think of
something before I see him again.”
In the car on the way to the deposition, the student overheard the lawyer make two calls on his cell
phone. The first call was to Linda Larue. It was clear from the lawyer’s side of the conversation that
the lawyer represented Linda’s husband in a divorce action and he was urging her to sign a settlement
agreement. He ended the call by saying, “Have your lawyer call me.”
The second call was to his receptionist. He said, “Tell Mrs. Smith I have decided to take her divorce
case, but she is hard to work with so I want at least $50 an hour more than my regular rate. No, on
second thought, write an engagement letter and tell her my fee will be $400 an hour. I’ll earn every
penny of that.”
The lawyer pulled into a parking place near the office where the deposition was to occur. He
commented as he got his briefcase out of the car, “This is a partition case, and I don’t know anything
about partition cases. I am on the slow side of this case, and I am just trying to stall.” When the deposi-
tion was over, the lawyer took the law student to lunch. The law student decided not to take the job.
Identify and discuss each violation of the ABA Model Rules of Professional Conduct.
PROFESSIONAL RESPONSIBILITY EXAM QUESTIONS 3.
Lou Lawyer represents Wanda Wife in a post-divorce proceeding in which the sole issue is Wanda’s
attempt to receive increased child support from Hal Husband. Lou is representing Wanda under a
contingent fee arrangement based on a percentage of the amount of increased child support Lou may
be able to obtain in the proceeding. Lou has never appeared before the judge presiding over the matter
and knows nothing about him.
Hal Husband is an attorney who is currently under investigation by the state disciplinary admin-
istrator based on a complaint from Candy Client, alleging that she and Hal had an affair during his
representation of her.
Lou called Candy Client and stated to Candy that she was investigating the disciplinary complaint
against Hal. Candy assumed Lou was from the disciplinary administrator’s office and told Lou all of
the details of her affair with Hal during the representation.
After a motion filed by Hal’s lawyer, the judge ruled that neither the disciplinary investigation nor
the alleged affair would have any relevance to the determination of child support and that no evidence
of either would be allowed.
Lou, furious at the judge’s ruling, directed her paralegal to prepare, sign, and file a motion to
disqualify the judge in the child support proceeding. The motion alleged that the judge was a woman-
hater who would do anything to protect deadbeat dads in domestic proceedings regardless of the
law and, therefore, should be disqualified from hearing Wanda’s child support matter. The motion
also included graphic details of the allegations of Candy Client regarding the alleged affair with Hal
Husband. Lou leaked a copy of the motion to the local newspaper, which published the contents of the
motion.
As Hal’s lawyer, you have learned all of the above facts.
Note: Your only assignment is to address possible ethics issues relating to Lou, not the issues in the
underlying child support case or Hal’s investigation.
You need not provide the citation to any rule, but must be able to discuss the content of each rule to
which you refer in your answer.
(1) Do you have any obligations under the ABA Model Rules of Professional Conduct? If so, what
are those obligations?
(2) Has Lou violated any rule or rules of the ABA Model Rules of Professional Conduct? If so,
provide your analysis and conclusion for each such violation.
4. PROFESSIONAL RESPONSIBILITY EXAM QUESTIONS
For the past 15 years, Craig has represented Trevor, the chief executive officer and founder of
Computer Chips, Inc. (“Computer Chips”), a large publicly traded computer company. Trevor also
develops real property, and Craig currently represents Trevor in a lawsuit regarding Trevor’s ownership
of real property in Anytown, USA. The lawsuit is close to being settled and, once settled, Trevor will
own the property on which he plans to build a 40-story high rise condominium.
Trevor has become frustrated with how the federal government is spending his “tax dollars.” One
day while Trevor is in Craig’s office to review the settlement documents regarding the lawsuit, Trevor
tells Craig that he intends to refuse to pay any more income taxes, because he believes that his taxes
should not go to support foreign countries. Craig thinks Trevor’s position is ridiculous and illegal,
and he wants to avoid any appearance that he endorses Trevor’s beliefs. Craig tells Trevor that he has
decided to terminate their relationship. Without Craig, the settlement of the lawsuit will collapse and
Trevor’s high rise project will be delayed for years.
After Trevor leaves the office, Craig contacts Harry, who is general counsel at Computer Chips, and
informs him of Trevor’s current decision not to pay his income taxes. It is Craig’s belief that Trevor
must be stopped, as Trevor’s actions will harm Computer Chips’s stock, in which Craig has invested
his life’s savings. Craig also tells Harry that he has begun selling his stock in Computer Chips and that
he believes it is his duty to tell the press about Trevor’s actions and to report Trevor’s potential crime to
the United States Attorney.
Before becoming general counsel of Computer Chips 15 years ago, Harry represented Trevor in
Trevor’s real estate holdings. Harry knows that Trevor can be a difficult client and that Trevor previ-
ously did not pay portions of his income tax in similar protest. Harry decides to call the chairman of
the board of Computer Chips and inform him of what Craig has divulged and of Trevor’s past actions
regarding the nonpayment of income taxes.
Have Craig and/or Harry violated any of the ABA Model Rules of Professional Conduct? Explain.
PROFESSIONAL RESPONSIBILITY EXAM QUESTIONS 5.
(1) After three years as an assistant district attorney, you decide to pursue your dream and open
your own law office. During your first month, you are contacted by a potential client who was involved
in an automobile accident. The case presents difficult issues regarding liability and insurance coverage.
Because of your inexperience in personal injury litigation, you contact a local lawyer experienced in
personal injury litigation to handle the claim. You, your client, and the other lawyer enter into a written
agreement which provides that (i) both lawyers will assume joint responsibility for the case, (ii) the
lawyers will receive a 35% fee based on the outcome of the case, and (iii) the lawyers will share the
fee.
Have your actions been appropriate under the ABA Model Rules of Professional Conduct? Explain
why or why not.
(2) A few weeks later, a longtime family friend advises you that he will soon be retiring from his
employment with a large insurance company. Although your friend has no legal education or training,
he has spent the last 20 years as an adjuster in the personal injury claims department. Your friend is
interested in part-time employment to avoid boredom, and you are excited about the opportunity to
expand your practice to include personal injury litigation. You agree to hire your friend as a legal assis-
tant (hereinafter referred to as your “legal assistant”) and to pay him an undetermined fee based on the
amount of the fee generated by each personal injury case.
Your legal assistant meets with prospective personal injury clients and gathers all relevant informa-
tion to support their claims. Since your criminal practice requires your frequent attendance in court,
you provide your legal assistant with blank, pre-signed letters of representation on your letterhead
which authorize your legal assistant to deal with the insurance companies in an effort to settle the case
and pre-signed letterhead stationery otherwise blank for his use in communicating with insurance
adjusters and clients regarding settlement negotiations. Many clients and insurance adjusters mistak-
enly believe your legal assistant is a lawyer.
After only two months on the job, your legal assistant advises you that he has successfully settled
your first personal injury case for $100,000. Your client is extremely satisfied and promises to refer
your firm to her friends. Your share of the settlement is $35,000. You pay your legal assistant $7,000
for a job well done.
Have your actions been appropriate under the ABA Model Rules of Professional Conduct? Explain
why or why not.
PROFESSIONAL RESPONSIBILITY EXAM ANSWERS 1.
Sam Stern (“Stern”) committed several violations of the ABA Model Rules of Professional Conduct
(“RPC”).
Specialization: Stern violated the RPC by having a sign on the door to his firm that reads, “Certi-
fied Divorce and Personal Injury Specialist.” A lawyer may not state or imply that he is certified as
a specialist in a particular field of law, unless: (i) the lawyer has been certified as a specialist by an
organization approved by an appropriate state authority or accredited by the ABA; and (ii) the name
of the certifying organization is clearly identified in the communication. Here, Stern stated that he was
a certified specialist in certain areas of the law, but failed to provide any supporting details. Thus, he
violated the RPC.
Supervising nonlawyers: Stern may have violated the RPC by allowing his receptionist to blatantly
discuss scandalous details of one of the firm’s divorce cases with persons outside of the firm. A lawyer
having direct supervisory authority over a nonlawyer must make reasonable efforts to ensure that the
person’s conduct is compatible with the professional obligations of the lawyer. Here, Stern was not
yet present when the receptionist was speaking on her cell phone about the divorce case. However, if
Stern knew the receptionist had a habit of discussing confidential information about his clients and did
nothing to remedy the situation, he has violated the RPC.
Diligence: Stern violated the RPC by failing to provide his clients with timely responses to their
messages. A lawyer must act with reasonable diligence and promptness in representing a client, and
his workload must be controlled so that each matter can be handled competently. Here, Stern’s client,
Mrs. Barnes, has apparently not received a response from Stern in months. This does not seem to be
an isolated incident, considering Stern also has an overflowing spindle of messages. Thus, Stern has
violated his duty of diligent representation.
Trust account funds: Stern violated the RPC by improperly taking money out of his firm’s client
trust account. Client funds must be kept in a separate trust account maintained in the state where the
lawyer’s office is situated, or elsewhere with the consent of the client or third person. A lawyer must
deposit into the trust account legal fees and expenses that were paid in advance, to be withdrawn only
as fees are earned or expenses are incurred. Here, Stern took money out of the trust account in order to
take the law student out to lunch, and this clearly violates the RPC.
Accepting appointments: Stern violated the RPC by attempting to avoid an appointment by Judge
Brown. A lawyer must not seek to avoid appointment by a tribunal to represent a person except for
good cause, such as: (i) representing the client is likely to result in a violation of the RPC or other law;
(ii) representing the client is likely to result in an unreasonable financial burden on the lawyer; or (iii)
the client or the cause is so repugnant to the lawyer as to be likely to impair the lawyer-client relation-
ship or the lawyer’s ability to represent the client. Here, Stern’s motivation for not wanting to serve as a
guardian ad litem in a divorce case was that it was “low paying.” This does not constitute good cause,
and therefore Stern has violated the RPC.
Dishonesty: In the incident above, Stern instructed his employee to lie to Judge Brown about
having a conflict that would not allow him to serve as a guardian ad litem. The RPC generally prohibit
conduct involving dishonesty, fraud, deceit, or misrepresentation. Thus, Stern’s dishonesty may have
violated the RPC.
Speaking with a represented person: Stern violated the RPC by speaking with his client’s spouse
regarding their divorce case. In the representation of a client, a lawyer must not communicate about the
subject of the representation with a person he knows to be represented by counsel in the matter unless
the other counsel has granted permission or he is otherwise authorized by law or court order to make
such direct communication. Here, Stern knew that his client’s wife, Linda Larue, was represented by
2. PROFESSIONAL RESPONSIBILITY EXAM ANSWERS
counsel because he said, “Have your lawyer call me.” Further, he was speaking with her about settling
the case, which related to the representation. Thus, Stern’s conversation with Linda violated the RPC.
Reasonableness of fee: Stern may have violated the RPC by charging Mrs. Smith an unreasonable
fee. A lawyer must not make an agreement for, charge, or collect an unreasonable fee or an unreason-
able amount for expenses. Among the factors to be considered in determining the reasonableness of a
fee are: (i) the time and labor required; (ii) the novelty and difficulty of the questions involved; (iii) the
skill required to perform the legal service; (iv) the likelihood, if apparent to the client, that acceptance
of the particular employment will preclude other employment; (v) the customary fee within the locality
for similar work; (vi) the amount involved and the result obtained; (vii) the nature and length of the
relationship between the parties; (viii) the experience, ability, and reputation of the attorney; (ix) the
time limitations imposed by the client or the circumstances; and (x) whether the fee is fixed or contin-
gent.
Here, Stern is planning to charge Mrs. Smith $400 an hour, which appears to be more than $50
higher than his normal hourly rate. There is no indication that the reason for this change is that the
legal work will be particularly complex, or that it will require more skill or preclude other employment.
Rather, Stern has increased the fee because he finds this client “hard to work with.” Despite his asser-
tion that he will “earn every penny” of the fee, this reasoning is suspect and indicates that the fee may
be unreasonable. Thus, Stern’s fee increase may violate the RPC.
Competence and duty to expedite litigation: Stern violated the RPC by failing to provide compe-
tent representation in the partition case. Once having entered into a lawyer-client relationship, a lawyer
must act competently and with care in handling legal matters for that client. Competent representation
requires the legal knowledge, skill, thoroughness, and preparation reasonably necessary for the repre-
sentation. A lawyer also has a duty to expedite litigation consistent with the interests of his client.
Here, Stern has been involved in the partition case for some time, as evidenced by his statement,
“I am on the slow side of this case and I am just trying to stall.” He has not spent his time educating
himself on this area of the law, since he still doesn’t “know anything about partition cases.” Thus, he
has violated the RPC by failing to provide competent representation. Furthermore, depending on the
interests of Stern’s clients, Stern also may have violated his duty to expedite litigation by intentionally
delaying the case.
(1) I have the ethical obligation to report Lou’s misconduct under the ABA Model Rules of Profes-
sional Conduct (“RPC”). Under the RPC, an attorney who knows that another lawyer has committed
a violation of the RPC that raises a substantial question as to that lawyer’s honesty, trustworthiness,
or fitness as a lawyer in other respects, must inform the appropriate professional authority. Disclosure
is not required if it requires the attorney to divulge confidential client information, but such is not the
case under these facts. Accordingly, if I am of the opinion that Lou’s conduct violated the RPC in any
way, I am ethically obligated to report the misconduct.
(2) Lou has committed several violations of the RPC. The first ethical issue concerns the contin-
gent fee arrangement based on a percentage of the amount of increased child support Lou may be
able to obtain in the proceeding. Under the RPC, a contingent fee is a fee that is dependent on the
successful resolution of a client’s case and payable from the judgment proceeds. Contingent fee
arrangements are permissible in some situations. However, a lawyer must not enter into a fee arrange-
ment for a domestic relationship matter in which the fee collected is contingent upon the amount of
alimony or support. Therefore, Lou’s contingent fee arrangement violated the RPC.
The second ethical issue concerns Lou’s communications with Candy Client. In representing a
client, a lawyer must not use means that have no substantial purpose other than to embarrass, delay,
or burden a third person; or use methods of obtaining evidence that violate the legal rights of such a
PROFESSIONAL RESPONSIBILITY EXAM ANSWERS 3.
person. Furthermore, in representing a client, a lawyer must not knowingly make a false statement of
material fact or law—or fail to disclose a material fact when disclosure is necessary to prevent misun-
derstanding—to a third person. In this case, Lou called Candy Client and stated to her that she was
investigating the disciplinary complaint against Hal. Lou should have disclosed to Candy Client that
she was not with the disciplinary administrator’s office to clarify any misunderstanding, and her failure
to do so violated the RPC.
The third issue concerns the unauthorized practice of law. The RPC prohibits a lawyer from
practicing law in a jurisdiction where doing so violates the regulation of the legal profession in that
jurisdiction or assisting another person in doing so. Here, when Lou authorized her paralegal to
prepare, sign, and file a motion to disqualify the judge in the child support proceeding, the paralegal
committed the unauthorized practice of law. Because Lou sanctioned that unauthorized practice, she
violated the RPC.
The fourth issue concerns the allegations made in the motion to disqualify the judge in the child
support hearing. A lawyer must not make a statement that the lawyer knows to be false or with reckless
disregard as to its truth or falsity concerning the qualifications or integrity of a judge. Here, even
though Lou knew “nothing” about the judge, he alleged that the judge was a woman-hater who would
do anything to protect deadbeat dads in domestic proceedings regardless of the law. While it is appro-
priate to file a motion to disqualify a judge for a legitimate reason, to state such outlandish, baseless
allegations is a violation of the RPC.
The fifth issue concerns the details of the alleged affair between Candy Client and Hal Husband
that Lou included in the motion to disqualify the judge in the child support hearing. A lawyer must
not knowingly disobey an obligation under the rules of a tribunal except for an open refusal based on
an assertion that no valid obligation exists. Here, the judge ruled that neither the disciplinary investi-
gation nor the alleged affair would have any relevance to the determination of child support and that
no evidence of either would be allowed. Accordingly, when Lou included graphic details of the allega-
tions of Candy Client regarding the alleged affair with Hal Husband in the motion, she violated the
RPC.
The sixth issue concerns the leaked copy of the motion that included graphic details of the alleged
affair between Candy Client and Hal Husband. A lawyer who is participating or has participated in
the investigation or litigation of a matter must not make an extrajudicial statement that the lawyer
knows or reasonably should know will be disseminated by means of public communication and will
have a substantial likelihood of materially prejudicing an adjudicative proceeding in the matter. Here,
Lou leaked a copy of the motion to the local newspaper, which published the contents of the motion.
Because of the graphic details of the alleged affair, Lou must have known of the prejudicial effect this
would have had on the proceeding and, therefore, violated the RPC.
Craig: Craig has violated the ABA Model Rules of Professional Conduct (“RPC”) by withdrawing
from representing Trevor, disclosing confidential information to Harry, and using confidential informa-
tion to the disadvantage of Trevor.
Withdrawal: Craig violated the RPC when he terminated his relationship with Trevor. At issue is
whether a lawyer may withdraw from representing a client. A lawyer may withdraw from representing
a client if withdrawal can be accomplished without material adverse effect on the interests of the client.
Here, Craig represents Trevor in a lawsuit that is close to being settled. The suit will not settle without
Craig, thereby delaying Trevor’s project for years. The multiyear delay will certainly have a material
adverse effect on Trevor. Therefore, Craig violated the RPC by terminating his relationship with
Trevor.
Craig may argue that he had to withdraw to avoid assisting Trevor with his crime of not paying
4. PROFESSIONAL RESPONSIBILITY EXAM ANSWERS
taxes. The RPC provide that a lawyer must not counsel or assist a client in conduct that the lawyer
knows is criminal or fraudulent. However, a lawyer may discuss a proposed course of conduct with a
client and explain to the client that the conduct would be unlawful. Here, Trevor is not asking Craig
to assist him in any way. He is merely telling Craig what he intends to do in the future. It is proper for
Craig to discuss this with Trevor and explain to him that it is illegal to not pay taxes. Continuing to
represent Trevor in the condominium lawsuit would not have assisted Trevor in his intended crime.
Finally, Craig may argue that he was justified in withdrawing from the representation of Trevor
because of his moral opposition to Trevor’s plan. A lawyer may withdraw if the client insists upon
taking action that the lawyer considers repugnant or with which the lawyer has a fundamental
disagreement. Here, Craig thought Trevor’s plan was “ridiculous and illegal.” However, it does not
seem that Trevor “insisted” on not paying taxes in the future—in fact, he made only one statement to
that effect, and there is no indication that Craig tried to talk him out of it. Given that the lawsuit was
about to settle, it is unlikely that Craig’s indignation was proper justification for withdrawing at the
last minute.
Disclosing information about Trevor: Craig violated the RPC when he disclosed confidential
information about Trevor to Harry. At issue is whether a lawyer may discuss confidential informa-
tion he received from a client with a third party. A lawyer must not reveal information relating to the
representation of a client. A lawyer may reveal such information, however, to prevent the client from
committing a crime or fraud that is reasonably certain to cause substantial financial loss to a person
if the client is using or has used the lawyer’s services to commit the crime or fraud. Here, Trevor told
Craig that he is no longer going to pay his taxes. This information was told to Craig in confidence,
but Craig then told Harry. Trevor does intend to commit tax fraud, which Craig believes could cause
substantial financial harm to the stockholders of Chips, Inc. However, there is no indication that Trevor
has used, or is using, Craig’s services in his plan. Thus, the RPC were violated when Craig gave Harry
this information.
Using information to Trevor’s disadvantage: Craig violated the RPC when he used confidential
client information to his client’s disadvantage by unloading the stock he owned in Computer Chips,
Inc. A lawyer must not use information relating to the representation of a client to the client’s disad-
vantage unless the client consents. Here, Craig has invested his life’s savings in Computer Chips,
Inc.’s, stock, and he has begun to sell that stock based on Trevor’s decision to not pay his income
taxes. As chief executive officer and founder of the company, Trevor will undoubtedly be harmed by
the sale of Craig’s stock, and it was unethical for Craig to use the information to his personal advan-
tage.
Harry: Harry violated the RPC when he divulged Trevor’s past tax crime and when he failed to
report Craig’s professional misconduct, but not when he divulged Trevor’s intent to commit the same
crime in the future.
Divulging information about Trevor: Harry violated the RPC by divulging Trevor’s past tax crime,
but not by divulging his intent to stop paying taxes again. At issue is whether a lawyer may divulge
information about a client.
A lawyer must not reveal information relating to the representation of a client, regardless of when
or where it was acquired. This duty of confidentiality continues to apply even after the lawyer-client
relationship has terminated. Here, Harry represented Trevor in his real estate holdings over 15 years
ago. Harry knows that Trevor previously has not paid his taxes. Even though Harry no longer repre-
sents Trevor, the duty of confidentiality still applies. Thus, Harry violated the RPC when he divulged
Trevor’s past tax crime.
It was proper for Harry to inform the board of Trevor’s intended crime. If a lawyer for an organiza-
tion knows that an officer of the organization intends to act in a matter related to the representation
that is likely to result in substantial injury to the organization, the lawyer must proceed as is reason-
ably necessary in the best interest of the organization. Here, Trevor is the CEO of Computer Chips
and intends to stop paying all income taxes. If Trevor does not pay income taxes related to Computer
PROFESSIONAL RESPONSIBILITY EXAM ANSWERS 5.
Chips, it will likely result in substantial injury to the organization. As general counsel for Computer
Chips, it is proper for Harry to inform the board of Trevor’s intended course of conduct.
Failure to report Craig’s violations of the RPC: Harry probably violated the RPC when he failed
to report Craig for divulging confidential client information to him. At issue is whether a lawyer must
report professional misconduct. A lawyer having knowledge that another lawyer has committed a
violation of the RPC that raises a substantial question as to that lawyer’s honesty, trustworthiness, or
fitness as a lawyer in other respects, must inform the appropriate professional authority. Here, Harry
knows that Craig has told him confidential information about a client. This is a violation of the RPC. It
probably raises a substantial question as to Craig’s fitness as a lawyer, because lawyer-client confidenti-
ality is a cornerstone of the legal profession. Therefore, Harry should have reported this misconduct to
the appropriate professional authority. His failure to do so constitutes a violation of the RPC.
(1) My actions in accepting representation of the client and sharing the fee with the other lawyer
have been appropriate under the ABA Model Rules of Professional Conduct (“RPC”); however, my
contingent fee agreement with the client was not in compliance with the RPC.
Competent representation: The RPC provide that a lawyer must act competently in handling a legal
matter for a client. Competent representation requires the legal knowledge, skill, thoroughness, and
preparation reasonably necessary for the representation. In determining whether a lawyer employs the
requisite knowledge and skill in a particular matter, a relevant factor is whether it is feasible to refer the
matter to, or associate with, a lawyer of established competence in the particular field. Here, I did not
have experience with personal injury litigation, so I associated with a lawyer who was so experienced.
Thus, by associating with an experienced personal injury lawyer, I can provide competent representa-
tion to the client, and I have not violated the RPC by accepting representation of the client.
Fee sharing: I also have not violated the RPC by entering into the written agreement with my
client and the other lawyer to share the fee with the other lawyer. The RPC provide that a lawyer may
ethically divide fees with an outside lawyer if the client consents and the division is in proportion to
the services performed by each lawyer, or by written agreement with the client, each lawyer assumes
joint responsibility for the representation. Here, there was a written agreement; the agreement provided
that both lawyers will assume joint responsibility for the representation; and the client entered into the
agreement, thereby consenting to it. Consequently, I did not violate the RPC by agreeing with my client
and the other lawyer to share the fee with the other lawyer.
Contingent fee: I did not violate the RPC by charging my client a 35% contingent fee in the matter;
but I did violate the RPC by not disclosing in the written agreement the information required by the
RPC. Generally, a contingent fee is permitted in a civil case if the fee is reasonable. In determining the
reasonableness of a fee, courts will consider such factors as the time and labor required for the repre-
sentation, the experience of the attorney, and the amount involved and the result obtained. Moreover,
a contingent fee agreement must be in a writing that states (i) the percentage(s) that accrue to the
lawyer(s) in the event of settlement, trial, or appeal; (ii) litigation and other expenses to be deducted
from the recovery; and (iii) whether such expenses are to be deducted before or after the contingent fee
is calculated.
Here, the contingent fee agreement was in a writing. The matter involves a personal injury claim,
which is a civil matter, so my charging a contingent fee was not prohibited by the RPC. Also, the 35%
fee based on the outcome of the case was not unreasonable under the circumstances. I associated an
experienced attorney to handle the matter, the case could very well go to trial and involve a great deal
of time and labor, and the case also could be worth a lot of money. However, the written agreement did
not specifically indicate the percentage(s) that would accrue to the lawyers in the event of settlement,
trial, or appeal; it only stated that the lawyers will receive a 35% fee based on the outcome of the case.
6. PROFESSIONAL RESPONSIBILITY EXAM ANSWERS
Furthermore, the agreement did not discuss the litigation and other expenses to be deducted from the
recovery and how they would be deducted. Therefore, the written agreement that I entered into with
the client to charge the contingent fee was not proper under the RPC.
(2) My actions in sharing my fees with my legal assistant and in allowing him to handle the
personal injury cases have not been appropriate under the RPC.
Fee sharing: The RPC provide that a lawyer must not share legal fees with a nonlawyer. Here, I
have agreed to pay my legal assistant an undetermined fee based on the fee that I receive for each
personal injury case. In fact, after my legal assistant settled my first personal injury case for $100,000,
of which my fee was $35,000, I paid my legal assistant $7,000. Thus, I agreed to (and did in fact) share
my legal fees with my nonlawyer legal assistant and thereby violated the RPC.
Unauthorized practice of law: My actions in allowing my legal assistant to deal with my clients
and the insurance companies and their claims adjusters also violate the RPC. The RPC provide that
a lawyer must not assist a person who is not a member of the bar in the performance of an activity
that constitutes the unauthorized practice of law. The practice of law includes the settling of cases for
clients. In this case, I have provided my legal assistant with blank, pre-signed letters and stationery
that authorize him to deal with my clients and the insurance companies (and their claims adjusters)
regarding settlement of my clients’ personal injury cases. In fact, some of my clients and the insurance
adjusters actually believe that my legal assistant is a lawyer. Thus, I have provided my nonlawyer legal
assistant with the authority to settle cases for my clients, which activity constitutes the practice of law.
Consequently, I have assisted my legal assistant in the unauthorized practice of law and have thereby
violated the RPC.
Supervising nonlawyers: In addition, by providing my legal assistant with control over my personal
injury cases, I have failed to properly supervise him. The RPC provide that a lawyer with direct
supervisory authority over a nonlawyer must ensure that the nonlawyer’s conduct is compatible with
the professional obligations of the lawyer. Here, I have cloaked my legal assistant with the necessary
authority to practice law although he is not a lawyer and his practice of law is prohibited by the RPC.
Therefore, I have not ensured that my legal assistant’s conduct is compatible with my professional
obligations under the RPC and so have violated the RPC.